r. m. m. - 24 · 2019. 11. 29. · draga tĂtucu mariana-romania claudia nĂnuȚi-romania dan...

101
ROMANIAN MATHEMATICAL SOCIETY Mehedinți Branch SPRING EDITION 2020 R. M. M. - 24 ROMANIAN MATHEMATICAL MAGAZINE ISSN 2501-0099

Upload: others

Post on 29-Oct-2020

25 views

Category:

Documents


4 download

TRANSCRIPT

Page 1: R. M. M. - 24 · 2019. 11. 29. · draga tĂtucu mariana-romania claudia nĂnuȚi-romania dan nedeianu-romania gabriela bondoc-romania ovidiu ticuȘi-romania romanian mathematical

ROMANIAN MATHEMATICAL SOCIETYMehedinți Branch

SPRING EDITION 2020

R. M. M. - 24 ROMANIAN MATHEMATICAL

MAGAZINE

ISSN 2501-0099

Page 2: R. M. M. - 24 · 2019. 11. 29. · draga tĂtucu mariana-romania claudia nĂnuȚi-romania dan nedeianu-romania gabriela bondoc-romania ovidiu ticuȘi-romania romanian mathematical

Romanian Mathematical Society-Mehedinți Branch 2020

1 ROMANIAN MATHEMATICAL MAGAZINE NR. 24

ROMANIAN MATHEMATICAL SOCIETY

Mehedinți Branch

ROMANIAN MATHEMATICAL MAGAZINE

R.M.M.

Nr.24-2020

Page 3: R. M. M. - 24 · 2019. 11. 29. · draga tĂtucu mariana-romania claudia nĂnuȚi-romania dan nedeianu-romania gabriela bondoc-romania ovidiu ticuȘi-romania romanian mathematical

Romanian Mathematical Society-Mehedinți Branch 2020

2 ROMANIAN MATHEMATICAL MAGAZINE NR. 24

ROMANIAN MATHEMATICAL SOCIETY

Mehedinți Branch

DANIEL SITARU-ROMANIA EDITOR IN CHIEF ROMANIAN MATHEMATICAL MAGAZINE-PAPER VARIANT

ISSN 1584-4897 GHEORGHE CĂINICEANU-ROMANIA

EDITORIAL BOARD

DAN NĂNUȚI-ROMANIA EMILIA RĂDUCAN-ROMANIA MARIA UNGUREANU-ROMANIA DANA PAPONIU-ROMANIA GIMOIU IULIANA-ROMANIA DRAGA TĂTUCU MARIANA-ROMANIA CLAUDIA NĂNUȚI-ROMANIA DAN NEDEIANU-ROMANIA GABRIELA BONDOC-ROMANIA OVIDIU TICUȘI-ROMANIA

ROMANIAN MATHEMATICAL MAGAZINE-INTERACTIVE JOURNAL ISSN 2501-0099 WWW.SSMRMH.RO

DANIEL WISNIEWSKI-USA EDITORIAL BOARD VALMIR KRASNICI-KOSOVO

ALEXANDER BOGOMOLNY-USA

Page 4: R. M. M. - 24 · 2019. 11. 29. · draga tĂtucu mariana-romania claudia nĂnuȚi-romania dan nedeianu-romania gabriela bondoc-romania ovidiu ticuȘi-romania romanian mathematical

Romanian Mathematical Society-Mehedinți Branch 2020

3 ROMANIAN MATHEMATICAL MAGAZINE NR. 24

CONTENT

Identities and inequalities in cyclic quadrilaterals- Claudia Nănuți, Daniel Sitaru............................4

Gakopoulos’ Lemmas- Thanasis Gakopoulos.............................................................................................9

About 1010 Inequality in Triangle – RMM 2018 – Marin Chirciu.......................................................15

About 996 Inequality in Triangle – RMM 2018 – Marin Chirciu.........................................................16

About 1070 Inequality in Triangle – RMM 2018 – Marin Chirciu.......................................................18

Properties of the eigenvalues of some classes of real matrices-Marian Ursărescu……………20 Structuri algebrice (I) - Vasile Buruiană…….……………………………………………………………………….23

Inequalities with cevians (I) – Bogdan Fustei….………………………………………………………………….29

Proposed problems………………………………………….………………………………………………………………...31

Index of proposers and solvers RMM-24 Paper Magazine.………………………………………….………100

Page 5: R. M. M. - 24 · 2019. 11. 29. · draga tĂtucu mariana-romania claudia nĂnuȚi-romania dan nedeianu-romania gabriela bondoc-romania ovidiu ticuȘi-romania romanian mathematical

Romanian Mathematical Society-Mehedinți Branch 2020

4 ROMANIAN MATHEMATICAL MAGAZINE NR. 24

IDENTITIES AND INEQUALITIES IN CYCLIC QUADRILATERALS By Claudia Nănuți, Daniel Sitaru – Romania

Abstract: In this paper are proved several metric identities in cyclic quadrilaterals Keywords and phrases: Bretschneider, Brahmagupta; Ptolemy; Girard. Notations: If is a cyclic quadrilateral denote:

= ; = ; = ; = ; = ; = ;Δ – area; – semiperimeter; , , , – exradii; – circumradii. By Bretschneider’s formula:

Δ = ( − )( − )( − )( − ) − cos (1)

+ = + = ; sin = sin ; sin = sin ; cos = − cos ; cos = − cos Replace + = in (1) and we obtain:

Δ = ( − )( − )( − )( − ) (2) which is called Brahmagupta’s formula.

Ptolemy’s theorem states that:

= + (3) Proposition 1: In any cyclic quadrilateral holds:

cos =( )

= − cos (4)

Proof: By the law of cosine in Δ ;Δ :

= + − 2 cos = + − 2 cos − − + = 2( cos − cos( − ))

2 cos ( + ) = − − +

cos =− − +2( + )

Proposition 2: In any cyclic quadrilateral holds:

sin = sin = (5)

sin = sin = (6) Proof:

Δ = [ ] + [ ] =12 sin +

12 sin ⇒

2Δ = sin + sin( − )

2Δ = ( + ) sin ⇒ sin =2+ = sin

Page 6: R. M. M. - 24 · 2019. 11. 29. · draga tĂtucu mariana-romania claudia nĂnuȚi-romania dan nedeianu-romania gabriela bondoc-romania ovidiu ticuȘi-romania romanian mathematical

Romanian Mathematical Society-Mehedinți Branch 2020

5 ROMANIAN MATHEMATICAL MAGAZINE NR. 24

Δ = [ ] + [ ] =12 sin +

12 sin

2Δ = sin + sin( − )

2Δ = sin ( + ) ⇒ sin =2Δ+ = sin

Proposition 3: In any cyclic quadrilateral holds:

cos 2 =( − )( − )

+ = sin 2 ; cos 2 =( − )( − )

+ = sin 2

cos = ( )( ) = sin ; cos = ( )( ) = sin (7) Proof:

cos 2 =1 + cos

2 =1 +

( )

2 =

=2( + ) + − − +

4( + ) =( + ) − ( − )

4( + ) =

=( + − + )( + + − )

4( + ) =(2 − 2 )(2 − 2 )

4( + ) =( − )( − )

+

Proposition 4: In any Δ cyclic quadrilateral holds:

tan 2 =( − )( − )( − )( − ) = cot 2 ; tan 2 =

( − )( − )( − )( − ) = cot 2

tan = ( )( )( )( )

= cot ; tan = ( )( )( )( )

= cot (8)

Proof:

tan 2 =sin

cos=

( )( )

( )( ) =( − )( − )( − )( − ) = tan

−2 = cot 2

Proposition 5: (GIRARD’S IDENTITY) In any cyclic quadrilateral holds:

( + )( + )( + ) = 16 Δ (9) Proof: In Δ : = ; In Δ : = By proposition (2):

⋅= ⇒ = (10)

⋅= ⇒ = (11)

By multiplying (10); (11):

=16Δ

( + )( + )

By Ptolemy’s theorem: = + , hence:

+ =16 Δ

( + )( + )

( + )( + )( + ) = 16 Δ

Page 7: R. M. M. - 24 · 2019. 11. 29. · draga tĂtucu mariana-romania claudia nĂnuȚi-romania dan nedeianu-romania gabriela bondoc-romania ovidiu ticuȘi-romania romanian mathematical

Romanian Mathematical Society-Mehedinți Branch 2020

6 ROMANIAN MATHEMATICAL MAGAZINE NR. 24

Proposition 6 (circumradii) In any cyclic quadrilateral holds:

= ( )( )( ) (12) Proof: By Girard’s identity:

( + )( + )( + ) = 16 Δ

=( + )( + )( + )

16Δ

=( + )( + )( + )

Proposition 7 (diagonals) In any cyclic quadrilateral holds:

=( + )( + )

+ ; =( + )( + )

+

Proof: By (10); (11):

=4Δ

+=( ) 1

+( + )( + )( + ) =

( + )( + )+

=4Δ

+ =( ) 1

+( + )( + )( + ) =

( + )( + )+

The circles tangent to a side of the cyclic quadrilateral and tangent to the extensions of its two other sides has radii , , , are called exradii. Proposition 8 (exradii) In any cyclic quadrilateral holds:

= =( )( )

(13), = =( )( )

=tan + tan

( − )( + ) , =tan + tan

( − )( + )

Proof: Denote – centre of excircle tangent to .

tan(∢ ) = = (14)

Page 8: R. M. M. - 24 · 2019. 11. 29. · draga tĂtucu mariana-romania claudia nĂnuȚi-romania dan nedeianu-romania gabriela bondoc-romania ovidiu ticuȘi-romania romanian mathematical

Romanian Mathematical Society-Mehedinți Branch 2020

7 ROMANIAN MATHEMATICAL MAGAZINE NR. 24

tan(∢ ) = tan = cot (15)

By (14); (15):

= cot ⇒ = tan (16)

tan(∢ ) = = (17)

tan(∢ ) = tan = cot (18)

By (17); (18):

= cot ⇒ = tan (19)

= = + =( );( )

tan 2 + tan 2 = tan 2 + tan 2 ⇒ =tan + tan

=tan + tan

Δ tan + Δ tan=

=( ) Δ

( − )( − )( − )( − ) ⋅ ( )( )( )( )

+ ( − )( − )( − )( − ) ⋅ ( )( )( )( )

( − )( − ) + ( − )( − ) =Δ

( − )(2 − − ) =Δ

( − )( + )

Proposition 9: In any cyclic quadrilateral holds:

+ + + ≥2

Proof: By (13): =

( )( ); =

( )( )

By adding:

+ = + = ⋅( )( )

= ⋅( )( )

(20)

Analogous:

+ = ⋅( )( )

(21)

By adding (20); (21):

+ + + =Δ+ ⋅

+( − )( − ) +

Δ+ ⋅

+( − )( − ) ≥

≥ Δ ⋅ 2 ( )( )( )( )( )( )( )( )

= Δ ⋅ = 2 (21)

Page 9: R. M. M. - 24 · 2019. 11. 29. · draga tĂtucu mariana-romania claudia nĂnuȚi-romania dan nedeianu-romania gabriela bondoc-romania ovidiu ticuȘi-romania romanian mathematical

Romanian Mathematical Society-Mehedinți Branch 2020

8 ROMANIAN MATHEMATICAL MAGAZINE NR. 24

+ + + = + + + ≥

≥+ + ++ + + ≥

( ) 22 =

2

Equality holds for = = = ( – square)

Proposition 10: In any cyclic quadrilateral holds:

+ + + ≥32

Proof: By (13):

= tan 2 + tan 2 ; = tan 2 + tan 2

= tan 2 + tan 2 ; = tan 2 + tan 2

By adding: + + + = 2 tan + tan + tan + tan (22)

Let be : 0, → ℝ; ( ) = tan ; ( ) =

( ) = = = > 0; – convexe

By Jensen’s inequality:

tan 2 + tan 2 + tan 2 + tan 2 ≥ 4 tan+ + +

2 =

= 4 tan = 4 tan = 4 tan = 4 (23) By (22); (23):

+ + + ≥ 2 ⋅ 4 = 8 (24)

+ + + = + + + ≥

≥+ + +

+ + + ≥( ) 8

2 =32

Equality holds for = = = ( – square) BIBLIOGRAPHY:

1. V.Pop,N.Minculete,M.Bencze-“An introduction in quadrilaterals geometry”-EDP-Bucharest-2015

2. Romanian Mathematical Magazine-Interactive Journal-www.ssmrmh.ro

Page 10: R. M. M. - 24 · 2019. 11. 29. · draga tĂtucu mariana-romania claudia nĂnuȚi-romania dan nedeianu-romania gabriela bondoc-romania ovidiu ticuȘi-romania romanian mathematical

Romanian Mathematical Society-Mehedinți Branch 2020

9 ROMANIAN MATHEMATICAL MAGAZINE NR. 24

GAKOPOULOS’ LEMMAS by Thanasis Gakopoulos-Greece

Lemma 1 Lemma 2

Plan 1

= = ⋅ ⋅ + 1 − ⋅ Lemma 1

= = ⋅ ⋅ + ⋅ Lemma 2

Proof. PLAGIOGONAL system: ≡ , ≡

(0,0), ( , 0), ( + , 0), ( , 0), (0, ), (0, ), ( , ), = , =

( , )

+ + = , + = , + =

: + = 1 (1), : = (2)

(1),(2)→ ( , )

=( + ) + ( + ) − ( + )

( + ) + −

=( + ) −

( + ) + −

=−− = ⋅ ⋅ + 1 − ⋅ →

→ = ⋅ ⋅ + 1 − ⋅

=−− =

1⋅

++ → =

1⋅ ⋅ + ⋅

Page 11: R. M. M. - 24 · 2019. 11. 29. · draga tĂtucu mariana-romania claudia nĂnuȚi-romania dan nedeianu-romania gabriela bondoc-romania ovidiu ticuȘi-romania romanian mathematical

Romanian Mathematical Society-Mehedinți Branch 2020

10 ROMANIAN MATHEMATICAL MAGAZINE NR. 24

Lemma 3 Lemma 4

Plan 2

= = ⋅ ⋅ or = = ⋅ ⋅ Lemma 3

= = ⋅ + ⋅ Lemma 4

Note: Lemma 4 is on the internet under the name Lemma Cristea. Proof: From plan 1, if ≡ then = 0 and we have Lemma 3. From Plan 1, if ≡ , then

= 0 and = . So, we have Lemma 4. Applications of Lemma 3 for the proof of theorems. Application 1

= ⋅ ⋅ → =

21

Application 2

= ⋅ ⋅ = ⋅ 1 ⋅ → =+

Page 12: R. M. M. - 24 · 2019. 11. 29. · draga tĂtucu mariana-romania claudia nĂnuȚi-romania dan nedeianu-romania gabriela bondoc-romania ovidiu ticuȘi-romania romanian mathematical

Romanian Mathematical Society-Mehedinți Branch 2020

11 ROMANIAN MATHEMATICAL MAGAZINE NR. 24

Application 3

=⋅ cos⋅ cos =

⋅=

− ++ −

= ⋅ cos =⋅

=2

+ −

= ⋅ ⋅ → =− ++ − ⋅

2+ −

Application 4

∥ → =

= ⋅ ⋅ → =

Application 5

∥ → = , =

= ⋅ ⋅ = ⋅ ⋅ → ⋅ ⋅ = 1

(Menelaus theorem: Δ , )

Page 13: R. M. M. - 24 · 2019. 11. 29. · draga tĂtucu mariana-romania claudia nĂnuȚi-romania dan nedeianu-romania gabriela bondoc-romania ovidiu ticuȘi-romania romanian mathematical

Romanian Mathematical Society-Mehedinți Branch 2020

12 ROMANIAN MATHEMATICAL MAGAZINE NR. 24

Application 6

, : = ⋅ ⋅

, : = ⋅ ⋅→(÷)

⋅ = ⋅ →

→ ⋅ ⋅ = 1 (Ceva theorem) Application 7

, : = ⋅ ⋅

, : = ⋅ ⋅→( )

→ 2 = ⋅ + ⋅ = ⋅+

+ ⋅+

→ 2 = + ⋅ + + ⋅ → 2 = + + + →

→ 2 = 2 + 2 → = + (Van Aubel theorem) Exercises that can be solved with the previous Lemmas.

Exercise 1.

= = . Find: , ,

Proposed by Thanasis Gakopoulos-Greece

Page 14: R. M. M. - 24 · 2019. 11. 29. · draga tĂtucu mariana-romania claudia nĂnuȚi-romania dan nedeianu-romania gabriela bondoc-romania ovidiu ticuȘi-romania romanian mathematical

Romanian Mathematical Society-Mehedinți Branch 2020

13 ROMANIAN MATHEMATICAL MAGAZINE NR. 24

Exercise 2. (Arsalan Waves)

: : = 1: 2: 3

: : = 3: 2: 1, = 3 . Find: Exercise 3 (Arsalan Waves)

: : = 3: 4: 3 : : = 4: 3: 4

= 160 Find: , ,

Exercise 4 (Romania 2009)

Δ , = , bisector of , bisector of , centroid of Δ

Prove: ∈ Exercise 5 (Proposed by Thanasis Gakopoulos-Greece)

Page 15: R. M. M. - 24 · 2019. 11. 29. · draga tĂtucu mariana-romania claudia nĂnuȚi-romania dan nedeianu-romania gabriela bondoc-romania ovidiu ticuȘi-romania romanian mathematical

Romanian Mathematical Society-Mehedinți Branch 2020

14 ROMANIAN MATHEMATICAL MAGAZINE NR. 24

Δ , ( ) = , ( ) = . Find: = ( , , ). If = ⋅ . Find:

Exercise 6 (Proposed by Thanasis Gakopoulos-Greece)

Δ , incenter of , ∥ , incenter of , , , , touch points

Find: , Exercise 7 (Proposed by Thanasis Gakopoulos-Greece)

Find:

= ( , , ), = ( , , )

Exercise 8 (internet)

= 90°, = , = , = 9, = 5

( ) =?

Page 16: R. M. M. - 24 · 2019. 11. 29. · draga tĂtucu mariana-romania claudia nĂnuȚi-romania dan nedeianu-romania gabriela bondoc-romania ovidiu ticuȘi-romania romanian mathematical

Romanian Mathematical Society-Mehedinți Branch 2020

15 ROMANIAN MATHEMATICAL MAGAZINE NR. 24

Exercise 9 (Proposed by Thanasis Gakopoulos-Greece)

= , = , = ∈ ℝ , 1 = 1 = 2 , = , =

Prove: = 1 + Exercise 10 (internet)

=

= , = Find:

ABOUT 1010 INEQUALITY IN TRIANGLE

ROMANIAN MATHEMATICAL MAGAZINE 2018

By Marin Chirciu – Romania 1) In Δ the following relationship holds:

⋅ℎ ≥ 2

486

Proposed by Daniel Sitaru – Romania Solution Using ≥ ℎ we obtain:

⋅ℎ ≥

⋅ ℎℎ = ℎ = 6 = 6

It suffices to prove that:

Page 17: R. M. M. - 24 · 2019. 11. 29. · draga tĂtucu mariana-romania claudia nĂnuȚi-romania dan nedeianu-romania gabriela bondoc-romania ovidiu ticuȘi-romania romanian mathematical

Romanian Mathematical Society-Mehedinți Branch 2020

16 ROMANIAN MATHEMATICAL MAGAZINE NR. 24

6 ≥ 2 ⇔ 3 ≥ ⇔ ≥ , which follows from Mitrinovici’s inequality

≥ 27 and Euler’s inequality ≥ 2 . Equality holds if and only if the triangle is equilateral. Remark Let’s find an inequality having on opposite sense: 2) In Δ the following relationship holds:

⋅ℎ ≤ 2 ( + )

Proposed by Marin Chirciu – Romania Solution

Using = cos , ℎ = and cos = ( ), we obtain:

∑ ⋅ = ∑ = ∑ ( )( )

= 8 ∑ ( )( )

(1)

From ( + ) ≥ 4 it follows: ∑ ( )

( )≤ ∑ ( ) = ∑ ( ) = ( ) = (2)

From (1) and (2) it follows ∑ ⋅ ≤ 2 ( + ).Equality holds if and only if the triangle is

equilateral. Remark The double inequality can be written: 3) In Δ the following inequality holds:

6 ≤⋅ℎ ≤ 2 ( + )

Solution See inequalities 1) and 2). Equality holds if and only if the triangle is equilateral.

ABOUT 996 INEQUALITY IN TRIANGLE ROMANIAN MAHTEMATICAL MAGAZINE

2018 By Marin Chirciu – Romania

1) In Δ the following relationship holds:

12 ≤+ℎ ≤

9 √3

Proposed by Mehmet Șahin – Ankara – Turkey Solution

We will prove the following lemma: Lemma 2) In Δ the following relationship holds:

+ℎ =

+ − 2

Proof:

Page 18: R. M. M. - 24 · 2019. 11. 29. · draga tĂtucu mariana-romania claudia nĂnuȚi-romania dan nedeianu-romania gabriela bondoc-romania ovidiu ticuȘi-romania romanian mathematical

Romanian Mathematical Society-Mehedinți Branch 2020

17 ROMANIAN MATHEMATICAL MAGAZINE NR. 24

Using ℎ = we obtain: +ℎ =

+=

( + )2 =

2 ( + − 2 )2 =

+ − 2

Using the Lemma, the left-hand inequality can be written:

≥ 12 ⇔ ≥ 14 − , true from Gerretsen’s inequality: ≥ 16 − 5 and Euler’s inequality ≥ 2 .

For right-hand inequality we prove that: ∑ ≤ ⇔ ≤ ⇔ ≤ 6 + 2 − , true from Gerretsen’s

inequality: ≤ 4 + 4 + 3 and Euler’s inequality ≥ 2 .

Then ≤ √ ⇔ ≤ √ (Mitrinovic’s inequality) Equality hold if and only if the triangle is equilateral. Remark If we replace ℎ with we propose: 3) In Δ the following relationship holds:

12 ≤+

≤ 42

− 1

Proposed by Marin Chirciu – Romania Solution We prove the following lemma: Lemma: 4) In Δ the following lemma:

+=

2( − 3 − 6 )

Proof Using ℎ = we obtain:

+=

+=

( + )( − )=

2 ( − 3 − 6 )=

2( − 3 − 6 )

Using Lemma, the left-hand inequality can be written:

≥ 12 ⇔ ≥ 12 + 3 , true from Gerretsen’s inequality:

≥ 16 − 5 and Euler’s inequality ≥ 2 . The right-hand inequality can be written:

2( − 3 − 6 )≤ 4

2− 1 ⇔

+ − 2≤

6⇔ ≤ 4 + 4 + 3

(Gerretsen’s inequality). Equality holds if and only if the triangle is equilateral.

Remark

Between the sums ∑ and ∑ the following relationship holds:

5) In Δ the following relationship holds:

Page 19: R. M. M. - 24 · 2019. 11. 29. · draga tĂtucu mariana-romania claudia nĂnuȚi-romania dan nedeianu-romania gabriela bondoc-romania ovidiu ticuȘi-romania romanian mathematical

Romanian Mathematical Society-Mehedinți Branch 2020

18 ROMANIAN MATHEMATICAL MAGAZINE NR. 24

+ℎ ≤

+

Proposed by Marin Chirciu – Romania Solution Using the above Lemmas, we have:

∑ = and ∑ = . We the write the inequality:

≤ ⇔ ≥ 10 + 7 , true from Gerretsen’s inequality: ≥ 16 − 5 and Euler’s inequality ≥ 2 .

Remark We can write the sequence of inequalities: 6) In Δ the following inequality holds:

12 ≤+ℎ ≤

+≤ 4

2− 1

Solution See inequalities 5), 3) and 1). Equality holds if and only if the triangle is equilateral.

ABOUT 1070 INEQUALITY IN TRIANLGE ROMANIAN MATHEMATICAL MAGAZINE

2019 By Marin Chirciu – Romania

1) In Δ , – centroid, – Lemoine’s point. Prove that:

⋅ + ⋅ + ⋅ ≥4√3

Proposed by Marian Ursărescu – Romania Solution We prove the following lemma: Lemma 2) In Δ , – centroid, – Lemoine’s point. Prove that:

⋅ =23 ⋅

( − 6 ) − (4 + )− (4 + )

Proof Using = and = ⋅ we obtain:

⋅ =23 ⋅

2+ + ⋅ =

43( + + ) ⋅ =

=⋅ [ ( )]

⋅ [ ( − 6 ) − (4 + ) ] = ⋅ ( )( )

, which follows

from the known identity in triangle ∑ ⋅ = − 6 − (4 + )

Back to the main problem: Using Lemma, the inequality can be written:

Page 20: R. M. M. - 24 · 2019. 11. 29. · draga tĂtucu mariana-romania claudia nĂnuȚi-romania dan nedeianu-romania gabriela bondoc-romania ovidiu ticuȘi-romania romanian mathematical

Romanian Mathematical Society-Mehedinți Branch 2020

19 ROMANIAN MATHEMATICAL MAGAZINE NR. 24

23 ⋅

( − 6 ) − (4 + )− (4 + ) ≥

4 √33

Taking into account Doucet’s inequality 4 + ≥ √3 it suffices to prove that:

⋅ ( )( )

≥ ( ) ⇔ ( − 2 − 14 ) + (4 + ) ≥ 0 (*)

We distinguish the following cases:

Case 1). If ( − 2 − 14 ) ≥ 0, the inequality is obvious.

Case 2). If ( − 2 − 14 ) < 0, the inequality can be rewritten:

(4 + ) ≥ (14 + 2 − ) which follows from Blundon-Gerretsen’s inequality 16 − 5 ≤ ≤ ( )

( )

It remains to prove that:

(4 + ) ≥(4 + )

2(2 − ) (14 + 2 − 16 + 5 ) ⇔ 2 − 3 − 2 ≥ 0 ⇔

( − 2 )(2 + ) ≥ 0, obviously from Euler’s inequality ≥ 2 . Equality holds if and only if the triangle is equilateral. Remark The inequality can be strengthened: 3) In Δ , – centroid, – Lemoine’s point. Prove that:

⋅ + ⋅ + ⋅ ≥43

(4 + )

Solution See (*) from the above proof.Equality holds if and only if the triangle is equilateral. Remark Inequality 3) is stronger than inequality 1). 4) In Δ , – centroid, – Lemoine’s point. Prove that:

⋅ + ⋅ + ⋅ ≥43

(4 + ) ≥4√3

Solution See 3) and Doucet’s inequality 4 + ≥ √3.Equality holds if and only if the triangle is equilateral. Remark Let’s find an inequality having an opposite sense: 5) In Δ , – centroid, – Lemoine’s point. Prove that:

⋅ + ⋅ + ⋅ ≤ 3(4 + )

Solution Using Lemma, the inequality can be written:

23 ⋅

( − 6 ) − (4 + )− (4 + ) ≤ 3

(4 + ) ⇔

Page 21: R. M. M. - 24 · 2019. 11. 29. · draga tĂtucu mariana-romania claudia nĂnuȚi-romania dan nedeianu-romania gabriela bondoc-romania ovidiu ticuȘi-romania romanian mathematical

Romanian Mathematical Society-Mehedinți Branch 2020

20 ROMANIAN MATHEMATICAL MAGAZINE NR. 24

⇔ (4 + + 12 − 2 ) ≥ (4 + ) ( − 2 ), which follows from

Gerretsen’s inequality: ( ) ≤ 16 − 5 ≤ ≤ 4 + 4 + 3 .

It remains to prove that:

(4 + )+ 4 + + 12 − 2 (4 + 4 + 3 ) ≥ (4 + ) ( − 2 ) ⇔

⇔ 3 − 8 + 6 − 4 ≥ 0 ⇔ ( − 2 )(3 − 2 + 2 ) ≥ 0, obviously from

Euler’s inequality ≥ 2 .Equality holds if and only if the triangle is equilateral.

Remark The double inequality can be written: 6) In Δ , – centroid, – Lemoine’s point. Prove that:

43

(4 + ) ≤ ⋅ + ⋅ + ⋅ ≤ 3(4 + )

Proposed by Marin Chirciu – Romania Solution See inequalities 3) and 5).Equality holds if and only if the triangle is equilateral. Remark. The following inequalities can be written: 7) In Δ , – centroid, – Lemoine’s point. Prove that:

12 ≤4√3

≤44

(4 + ) ≤ ⋅ + ⋅ + ⋅ ≤ 3(4 + ) ≤

32

Solution See inequalities 4), 6), Euler’s inequality ≥ 2 and Mitrinovic’s inequality ≥ 3 √3. Equality holds if and only if the triangle is equilateral. Reference: 1. Marian Ursărescu, 1070 Inequality in triangle, Romanian Mathematical Magazine, February 2019. 2. Marin Chirciu, Algebraic Inequalities, from beginner to performer, Paralela 45 Publishing House, Pitești, 2014. 3. Marin Chirciu, Inequalities with important lines in triangle from beginner to performer, Paralela 45 Publishing House, Pitești, 2018.

PROPERTIES OF THE EIGENVALUES OF SOME CLASSES OF REAL MATRICES

By Marian Ursărescu – Romania

In this article we will prove some interesting properties about the eigenvalues of some real matrices, followed by applications. For start, we will remember some of the classic results from matrices theory and their determinants. Definition

Page 22: R. M. M. - 24 · 2019. 11. 29. · draga tĂtucu mariana-romania claudia nĂnuȚi-romania dan nedeianu-romania gabriela bondoc-romania ovidiu ticuȘi-romania romanian mathematical

Romanian Mathematical Society-Mehedinți Branch 2020

21 ROMANIAN MATHEMATICAL MAGAZINE NR. 24

Let be ∈ (ℝ) and ∈ , (ℝ). If it exists ∈ ℂ such that = , then is called

own vector, and eigenvalue for the matrix .

Observation. The matricial equation ( − ) = is equivalent with the system:

(1)

( − ) + + ⋯+ = 0+ ( − ) + ⋯+ = 0

… … … … … … … … … … … … … … … … … .+ + ⋯+ ( − ) = 0

The system (1) has nonzero solutions ⇔ det( − ) = 0.

Definition The polynom ( ) = det( − ) is called characteristic polynom of the matrix , and the equation ( ) = det( − ) = 0 is called characteristic equation of the matrix . Observation The eigenvalues of matrix are the solutions of the characteristic equation. Theorem. The characteristic polynom has the expression ( ) = − Δ + Δ +⋯+ (−1) Δ , where Δ represents the sum of the principal minors having the order of the matrix − . Observation

1. … = det 2. + + ⋯+ =

3. + + ⋯+ = ∗ Cayley – Hamilton Theorem If ( ) is the characteristic polynom of matrix , then ( ) = . Definitions Let be ∈ (ℝ) 1. Matrix is called symmetric if: = 2. Matrix is called antisymmetric if = − . 3. Matrix is called orthogonal if: ⋅ = (we have denoted = transposed matrix). Theorem 1 The eigenvalues of some real symmetric matrix are real. Proof Let be ∈ (ℝ) with = . Let’s suppose that it exists an eigenvalue such that:

∈ ℂ ⇒ = (1)

We multiply to the left relation (1) with ⇒ = ⋅ (2)

We conjugate relation (1) ⇒ = (3) ∈ (ℝ)

We multiply to the left relationship (3) with ⇒ = ⋅ ⇒

⇒ ( ) = ( ⋅ ) ⇒ = ⋅ (4)

From (2) and (4) ⇒ ⋅ = ⋅ ⇒

− ( ⋅ ) = ⋅ = ⋅ + + ⋯+ ⋅ = ( ) + ( ) + ⋯+ ( ) > 0

Page 23: R. M. M. - 24 · 2019. 11. 29. · draga tĂtucu mariana-romania claudia nĂnuȚi-romania dan nedeianu-romania gabriela bondoc-romania ovidiu ticuȘi-romania romanian mathematical

Romanian Mathematical Society-Mehedinți Branch 2020

22 ROMANIAN MATHEMATICAL MAGAZINE NR. 24

⇒ − = 0 ⇒ = ⇒ ∈ ℝ.

Theorem 2 The eigenvalues of some real antisymmetric matrix are or nonzero or purely imaginary. Proof Let ∈ (ℝ) with = − . For start let’s prove that the only eigenvalues are nonzero. Let ∈ ℝ be an eigenvalue ⇒ = ⇒ by multiplying to the left with ⇒

= (1) ⇒ ( ) = ( ) ⇒ − = (2)

From (1)+(2)⇒ = − ⇒ 2 ⋅ = 0 ⇒ = 0

Let ∈ ℂ be an eigenvalue ⇒ = ⇒ = (3) ⇒ = ⇒ = ⋅ ⇒ ( ) = ( ⋅ ) ⇒

− = ⋅ (4) From (3)+(4)⇒ ⋅ = − ⋅ ⇒

+ ⋅ = 0 ⇒ + = 0 ⇒ = − ⇒ is purely imaginary, namely =

Theorem 3 The eigenvalues of an orthogonal real matrix have an absolute value equal with 1. Proof. Let be ∈ (ℝ) with = . Let be an eigenvalue ⇒ = ⇒ by multiplying to the left with ⇒ = (1)

= ⇒ = ⇒ = ⇒ by multiplying to the left with ⇒ = ⇒ ( ) = ( ⋅ ) ⇒

⇒ = ⋅ (2) (1) + (2) ⇒ ⋅ ⋅ ⋅ = | | ⋅ ( ⋅ )

= | | + | | + ⋯+ | | ⇒

⇒ ⋅ ⋅ = = | | ( ⋅ ) ⇒ ( ⋅ ) = | | ( ⋅ ) ⇒ | | = 1 ⇒ | | = 1

Applications 1. Let ∈ (ℝ) an antisymmetric matrix a) If is odd, then det = 0 b) If is even, then det is a perfect square. Proof a) = 2 + 1 ⇒ ( ) has an odd number of pairs ⇒ at least one is real ⇒ from theorem 2 ⇒ the real root is ⇒ det = … = 0. b) = 2 ⇒ ( ) has an even number of pairs, ( ) ∈ ℝ[ ] ⇒ the roots are complex conjugate, from theorem 2 ⇒ = and = − , = and = − … = and = − ⇒ det = … = ( ⋅ … ) > 0 2. Let ∈ (ℝ) such that = and = 0. Prove that: | | = 3

(Mathematical Gazette) Proof Let , , be the eigenvalues of matrix from theorem 3 ⇒ | | = | | = | | = 1 = 0 ⇔ + + = 0 ⇒ + + = 3 (known identity)⇒

| + + | = 3| || || | ⇒ ( ) = 3

Page 24: R. M. M. - 24 · 2019. 11. 29. · draga tĂtucu mariana-romania claudia nĂnuȚi-romania dan nedeianu-romania gabriela bondoc-romania ovidiu ticuȘi-romania romanian mathematical

Romanian Mathematical Society-Mehedinți Branch 2020

23 ROMANIAN MATHEMATICAL MAGAZINE NR. 24

3) Let be ∈ (ℝ). If ⋅ = and = 0 ⇒ ∈ {−2,0,2} (Mathematical Gazette)

Proof Let be , , the eigenvalues of matrix . From theorem 3⇒ | | = | | = | | = 1

= + + , = + + = 0 det( ⋅ ) = det ⇒ (det ) = 1 ⇒ det = ±1

+ + = 0 ⇒ ( + + ) = 2( + + )| | = 1 ⇒ ⋅ = 1, = 1,3

( + + ) = 21

+1

+1

( + + ) = 2 + + ⇒ ( ) = 2 det ; ∈ ℝ ⇒ = ⇒ ( − 2 det ) = 0 ⇒ or = ±2

STRUCTURI ALGEBRICE (I)

By Vasile Buruiană – Romania INTRODUCERE

Fie ( ,⋅) un semigrup comutativ unitar cu unitatea . Se spune că ∈ se divide (în ) la

∈ și se notează „ ⋮ ” dacă există ∈ astfel încât = . Se mai spune în acest caz că

e este multiplu al lui sau că este divizor (factor) al lui sau că divide notând în acest

ultim caz „ | ”. Evident că ∀ ∈ avem | (divizibilitatea este reflexivă) și dacă | și |

pentru , , ∈ atunci | (divizibilitatea este tranzitivă), deci relația de divizibilitate în

este o relație de preordine. Se spune că ∈ este inversabil sau unitate ⇔ ∃ ∈ și

= . Pentru , ∈ se spune că sunt asociate și notăm „ ∼ ” ⇔ ( | și | ). Se

spune că ∈ ∖ { } este atom ⇔ ( | , ∈ ⇒ ori = ori = ), iar ∈ − { } se

numește prim dacă din relația | ⇒ | sau | . Un element ∈ ∖ { } se numește

ireductibil sau idecompozabil dacă din | ⇒ ori ∼ ori este unitate, iar ∈ ∖ { } se

numește reductibil sau decompozabil dacă există ∈ , ≁ și neinversabil astfel că | .

Pentru , ∈ se numește cmmdc al lor (dacă există) un element ∈ , notat = ( , )

având proprietățile: | , | și ∀ ∈ cu | , | ⇒ | , la un cmmmc al lor (dacă

există) se numește un element ∈ având proprietățile ⋮ , ⋮ și ∀ ∈ cu

⋮ , ⋮ ⇒ ⋮ . Asemănător, acestea se definesc și pentru mai mult de două

elemente. Un semigrup ( ,⋅) comutativ unitar cu unitatea se numește atomic

⇔[(∀ ∈ ⇒ există o descompunere finită = … , ≥ 0 și atomi pentru

Page 25: R. M. M. - 24 · 2019. 11. 29. · draga tĂtucu mariana-romania claudia nĂnuȚi-romania dan nedeianu-romania gabriela bondoc-romania ovidiu ticuȘi-romania romanian mathematical

Romanian Mathematical Society-Mehedinți Branch 2020

24 ROMANIAN MATHEMATICAL MAGAZINE NR. 24

= 1, … , ) și ( = 0 ⇔ = ) și descompunerea este unică in afara ordinii și asocierii

factorilor]. Un semigrup ( ,⋅) se numește cu simplificarea ⇔[ ∀ , , ∈ și = ⇒

= ]. Dacă este inel integru comutativ și unitar, este evident că ( ∗ = ∖ {0},∘) este

semigrup cu simplificare și că | , | ⇒ |( ± ). O mulțime ⊂ se numește ideal

⇔[∀ , ∈ ⇒ − ∈ și ∀ ∈ , ∈ ⇒ ∈ ]. Idealul = = { | ∈ } se

numește principal (generat de ). Idealul generat de o mulțime constă din intersecția

tuturor idealelor lui care îl conțin pe și constă din toate sumele finite ∑ cu

∈ , ∈ . Notăm ( , … ) = ∑ idealul generat de = { , … , }. Un inel

integru comutativ unitar se numește euclidian ⇔ ∃ : ∗ → ℕ astfel că ∀ , ∈ ∗ cu

≠ 0,∃ , ∈ astfel că = + și ori = 0 ori ( ) < ( ). Un inel integru comutativ

și unitar se numește principal ⇔ orice ideal al său este principal.

Exemple și observații.

1) (ℕ∗,⋅) este semigrup cu simplificarea, singurul element inversabil fiind 1. In acest

semigrup relația de divizibilitate este și antisimetrică (deci de ordine). Intr-adevăr, dacă

, ∈ ℕ∗ și | , | ⇒ ∃ , ∈ ℕ∗ astfel ca = , = ⇒ = deci = 1 ⇒

= 1 = deci = .

2) Dacă , ∈ ℕ∗, și | ⇒ 1 ≤ ≤ . Într-adevăr dacă = , ∈ ℕ∗ ⇒ ≥ 1. Dacă

= 1 ⇒ = sau ≥ 2 fie ∈ ℕ∗ astfel ca succesorul său = + 1 = ⇒

= = = ( + 1) = + ≥ , deci 1 ≤ ≤

3) Dacă ∈ ℕ∗; > 1 atunci:

este reductibil ⇔[∃ , ∈ ℕ∗, 1 < < , 1 < < și = .

Demonstrație: „⇒” Dacă este reductibil ⇒ ∃ ≠ 1, ≠ și | . Fie = ⇒ 1 ≤ ≤

și 1 ≤ ≤ și 1 < < . Dar = 1 ⇒ = , iar dacă = ⇒ = 1, ceea ce este

imposibil.

„⇐” Fie = , 1 < < , 1 < < . Evident nu avem niciuna din relațiile ∼ , ∼ ,

inversabil, inversabil ⇒ este reductibil.

4) Dacă ∈ ℕ∗ , > 1 ⇒ ∃ un divizor ireductibil al lui . Intr-adevăr, luând

= { ∈ ℕ∗| | , > 1}, avem că ≠ ∅ căci | ⇒ ∃ = cel mai mic elemnt al lui ,

> 1. Dacă ar fi reductibil și ar exista astfel că | , 1 < < . Deci | | , < ,

Page 26: R. M. M. - 24 · 2019. 11. 29. · draga tĂtucu mariana-romania claudia nĂnuȚi-romania dan nedeianu-romania gabriela bondoc-romania ovidiu ticuȘi-romania romanian mathematical

Romanian Mathematical Society-Mehedinți Branch 2020

25 ROMANIAN MATHEMATICAL MAGAZINE NR. 24

1 < ⇒ ∈ ceea ce contrazice minimalitatea lui ⇒ este ireductibil și de aici

afirmația.

5) În (ℕ∗,⋅) sunt echivalente afirmațiile:

a) este prim; b) este ireductibil

Demonstrație: a) ⇒b). Dacă prin absurd ar fi prim și reductibil ar rezulta că ∃ , ∈ ℕ∗

astfel încât = , 1 < < , 1 < < . Cum = ⋅ 1 = ⇒ | ⇒ | sau

| ⇒ ≤ sau ≤ , ceea ce este contradictoriu

b)⇒ a) Presupunem prin absurd că există ∈ ℕ∗ ireductibil și cu proprietatea că | și

∤ , ∤ ; alegem , astfel ca sa fie minimal cu proprietatea de mai sus. Evident ≠ 1

căci atunci | . Rezultă că 1 < . Dacă am avea ≥ ⇒ ℕ

.î ∃ , ∈ ℕ astfel ca

= + , 0 < < , deci ∤ și deci și , deoarece = + , au proprietatea că

| , ∤ , ∤ ; însă < cee ace contrazice minimalitatea lui . Vom avea deci

1 < < (și analog 1 < < ). Dacă ∈ ℕ∗ și = nu putem avea = 1. Deci

> 1 ⇒ ∃ ireductibil, | . Cum = < ⋅ = ⇒ < deci ≤ < și cum

| iar era ireductibil minimal ce nu era prim ⇒ | sau | . Fie = , =

(admițând că | ). Din = ⇒ = , deci | și ∤ , ∤ , < ⇒ <

și iarăși se contrazice minimalitatea lui . Se infirmă astfel existența numerelor naturale

ireductibile care nu sunt prime.

6) Teorema fundamentală a aritmeticii

∀ ∈ ℕ∗ , > 1 ⇒ există , , … , prime (ireductibile) unice în afara unei permutări,

astfel că = , … ,

Demonstrație: Existența Fie = { | > 1, ≠ } ≠ ∅

⇒ ∃ = cel mai mic element al lui . Evident elementele lui nu sunt prime (ireductibile)

⇒ nu este ireductibil ⇒ este decomposibil ⇒ ∃ , ∈ cu = și 1 < < , 1 < < .

Cum , ∉ căci este minimal ⇒ și pot fi reprezentate ca produse finite elemente

prime, deci și = , ceea ce este absurd. Rămâne deci adevărată afirmația existenței.

Prima demonstrație a unicității (Euclid): Fie = … = … un element cu două

descompuneri în factori primi. Cum | … ⇒ există și îl putem presupune pe = ,

Page 27: R. M. M. - 24 · 2019. 11. 29. · draga tĂtucu mariana-romania claudia nĂnuȚi-romania dan nedeianu-romania gabriela bondoc-romania ovidiu ticuȘi-romania romanian mathematical

Romanian Mathematical Society-Mehedinți Branch 2020

26 ROMANIAN MATHEMATICAL MAGAZINE NR. 24

astfel că | . Datorită ireductibilității factorilor ⇒ = ⇒ … = … (1).

Dacă = 1; ≥ 2 atunci , … , sunt inversabile, ceea ce este absurd ⇒ = 1.

Deci pentru un număr > 1 care are o descompunere cu = 1 factori primi, afirmația este

adevărată. Folosim inducția după , și presupunem că afirmația este adevărată pentru

numerele > 1 având o descompunere cu − 1 factori. Din (1), având o astfel de

descompunere ⇒ − 1 = − 1 ⇒=ș

= , = 1, … , deci afirmația este adevărată și

pentru .

A doua demonstrație a unicității (Zermelor, Hasse, Lord Cherwell)

Evident 2 și 3 admit o descompunere unică. Dacă ∃ = … = … cel mai mic

număr cu două descompuneri, putem presupune ≤ ⋯ ≤ , ≤ ⋯ ≤ . Nu putem

avea existența unor , cu = , căci atunci = < ar avea două descompuneri

distincte și s-ar contrazice minimalitatea lui . Fie < și fie = … <

… = . Cum | , | ⇒ | − = ( − ) … = . Cum evident

∤ − și ∤ … ⇒ mai are o descompunere în care nu apare . Dar <

contrazice iarăși minimalitatea lui cu două descompuneri, fiind astfel infirmată existența

numerelor naturale cu două descompuneri distincte în factori ireductibile (primi). Deci orice

număr natural are o descompunere unică.

7) Rezultă din cele de mai sus că (ℕ∗,⋅) este un semigrup atomic în care atomii sunt chiar

elementele prime (ireductibile). El are o infinitate de atomi (dacă , … , are fi toate

numerele prime două câte două distincte, atunci … + 1 nu se divide prin minimul

dintre , … , și este distinct de fiecare dintre … deci = … + 1 este de

asemenea prim, ceea ce este contradictoriu)

8) ({ }, ≥ 0,⋅) unde este prim în ℕ∗ este un semigrup atomic cu un singur atom, iar

({1},⋅) este și el semigrup atomic fără atomi. (ℕ, +) este de asemenea semigrup atomic cu

singurul atom 1, singurul element inversabil fiind 0. Ultimele două semigrupuri atomice sunt

izomorfe prin ( ) = . Cu (ℕ, +) relația | echivalează cu ≤ .

9) Fie mulțimea atomilor din (ℕ∗,⋅). În locul scrierii unice = … , dacă factorii identici

se grupează sub același exponent vom mai scrie = … sau = ∏ ( )∈ unde

Page 28: R. M. M. - 24 · 2019. 11. 29. · draga tĂtucu mariana-romania claudia nĂnuȚi-romania dan nedeianu-romania gabriela bondoc-romania ovidiu ticuȘi-romania romanian mathematical

Romanian Mathematical Society-Mehedinți Branch 2020

27 ROMANIAN MATHEMATICAL MAGAZINE NR. 24

( ) == 1, … ,

dacă = și ( ) = 0 pentru ≠ , = 1, … , . Adică = ∏ ( )∈ unde

( ) sunt unici și aproape toți, cu excepția unui număr finit, sunt nuli. Dacă

= ∏ ( )∈ ; = ∏ ( )

∈ , = ∏ ( ) ( )∈ și avem și

| ⇔ ( ) ≤ ( )∀ ∈ . Ultima echivalență rezultă din aceea că dacă = cu

= ∏ ( )∈ avem ∏ ( ) = ∏ ( ) ( ) ⇔ ( ) = ( ) + ( ) ⇔

( ) ≤ ( );∀ ∈ .

10) Dacă = … este descompunerea unică, atunci numărul divizorilor lui este

( ) = ( + 1)( + 1) … ( + 1), după cum rezultă din observația precedentelor.

Evident este prim (atom, ireductibil)⇔ ( ) = 2 ⇔[ se divide doar prin și prim 1]

11) Fie ∈ ℕ∗, > 1. Atunci este prim ⇔ ∑ − = 2, unde [ ] este partea

întreagă a lui , adică cel mai mare întreg din stânga lui sau care coincide cu .

Într-adevăr, dacă | ⇒ ∃ ∈ ℕ cu = ⇔ = deci = iar − 1 = − 1 =

( − 1) + ( − 1) și deci = − 1 ⇒ − = − ( − 1) = 1. Dacă însă

∤ , există , ∈ ℕ astfel că = + , 0 < < , deci = și

− 1 = + ( − 1) ⇒ = deci − = − = 0. Deci

− = 1 ă |0 ă ∤ ⇒ ∑ − = numărul divizorilor lui = ( ). Deja

am văzut că este prim ⇔ ( ) = 2, de unde afirmația făcută.

12) ∀ , ∈ (ℕ∗,⋅) ⇒ ∃ cmmdc și ∃ cmmmc al lor. Într-adevăr doar = … ,

= … luănd = … și = … unde = min( , ) , = 1, … , ,

= max( , ) , = 1, … și o simplă verificare o a definițiilor arată că = ( , ),

= [ , ]. Observăm doar | ⇒ ( , ) = și [ , ] = , iar dacă ∤ putem căuta

masura lor comună maximă, adică cmmdc, luănd = + , 0 < < și constatînd că

( , ) = ( , ). Repetând procesul pentru și , rezultă algoritmul lui Euclid cu care putem

găsi cmmdc: scriem șirul de împărțiri cu rest

= + , 0 < <

= + , 0 ≤ <

= + , 0 ≤ <

Page 29: R. M. M. - 24 · 2019. 11. 29. · draga tĂtucu mariana-romania claudia nĂnuȚi-romania dan nedeianu-romania gabriela bondoc-romania ovidiu ticuȘi-romania romanian mathematical

Romanian Mathematical Society-Mehedinți Branch 2020

28 ROMANIAN MATHEMATICAL MAGAZINE NR. 24

ș.a.m.d. până ce obținem obligatoriu un rest nul (aceasta se va întâmpla într-un număr finit

de pași fiindcă > > ⋯ nu poate fi infinit). Atunci ( , ) va fi ultimul rest nenul din acest

algoritm și în plus [ , ] =( , )

.

13) ℤ este inel euclidian. Intr-adevăr, mai întâi arătăm că ∀ , ∈ ℤ, ≠ 0 ⇒ ∃ , ∈ ℤ, unice

astfel că = + , 0 ≤ < | | (teorema împărțirii cu rest în ℤ)

Existența: Pentru = 0 luăm = = 0. Pentru ≠ 0 și > 0, luăm mulțimea

= { − | ∈ ℤ} și în ea pe = − = cel mai mic element pozitiv. Aceste și =

verifică afirmația. Într-adevăr nu putem avea = − ≥ , căci ar rezulta

− ( + 1) > 0 și cum − ( + 1) < − = se contrazice minimalitatea lui

⇒ 0 ≤ < . Procedăm asemănător și pentru < 0. În privința unicității:

dacă = + = +

0 ≤ < | |0 ≤ < | | și ≠ , ≠ ⇒ | || − | = | − | și cum

| − | ≥ 1 ⇒ | | ≤ | − | ceea ce este absurd.

Luând în fine : ℤ∗ → ℕ pun ( ) = | |, cu cele de mai sus rezultă imediat că ℤ este

euclidian.

14) În inelul ℤ sunt adevărate afirmațiile:

a) ∀ , ∈ ℤ∗, idealul = { + | , ∈ ℤ} este principal și generatorul acestuia, , este

cmmdc al numerelor , .

b) (Lema Euclid) | și ( , ) = 1 ⇒ | . Elementele ireductibile de ℤ sunt prime.

c) ∀ ∈ ℤ se scrie unic sub forma = (−1) ( ) ∏ ( )∈ unde este mulțimea

elementelor prime pozitive din ℤ, ( ) = 0, > 01, < 0 și ( ) = 0 cu excepția

eventual a unui număr finit de elemente din ( ∈ , ∈ ℤ).

Demonstrație: a) Fie = + cel mai mic elemnt pozitiv din . Evident ∈ . Luând

apoi ∀ ∈ ⇒ ∃ , unice cu = + cu 0 ≤ < . Dar 0 < < cum = − ∈

se contrazice minimalitatea lui ⇒ = 0 deci ∈ ⇒ ⊂ ⊂ ⇒ = ℤ. Apoi cum

| ⋅ 1 + , | ⋅ 0 + 1 ⇒ | , iar dacă | , ⇒ | + = ⇒ =cmmdc. În

particular | , | = 1 ⇔ ∃ , ∈ ℤ cu + = 1

b) Cum ( , ) = 1 ⇒ ∃ , cu + = 1 ⇒ | + = și afirmația este dovedită

două luăm ireductibil, | și ∤ căci atunci ( , ) = 1 ⇒ | deci este prim.

Page 30: R. M. M. - 24 · 2019. 11. 29. · draga tĂtucu mariana-romania claudia nĂnuȚi-romania dan nedeianu-romania gabriela bondoc-romania ovidiu ticuȘi-romania romanian mathematical

Romanian Mathematical Society-Mehedinți Branch 2020

29 ROMANIAN MATHEMATICAL MAGAZINE NR. 24

c) Observăm mai întâi că ∀ > 0 și ∀ prim pozitiv ⇒ ∃ , unic astfel că | și ∤ .

Aceasta este evident căci puterile lui sunt crescătoare. Numărul ∈ ℕ, unic, se numește

ordinul lui în și se notează ( ) și verifică proprietățile ( ) = 0 ⇔ ∤ și

( ) = ( ) + ( )

Existența scrierii lui rezultă din teorema fundamentală a aritmeticii aplicată lui sau − .

Cum în ℤ∗ singurele unității sunt ±1, elementele asociate sunt egale sau diferă pun semn și

unicitatea rezultă aplicând funcția lui = (−1) ( ) ∏ ( ) și găsind că

( ) = ( ). Remarcăm că (ℤ∗,⋅) este semigrup cu simplificarea, dar nu este atomic

fiindcă orice element are divizorii ±1, ± .

INEQUALITIES WITH CEVIANS (I)

By Bogdan Fustei-Romania In ∆ABC holds:

2 ( − )( − ) ≤ ( − ) + ( − ), −semiperimeter

p-b+ p-c=a; ( − )( − ) = hence 4 ≤ (and analogous).

By =( -r)( + ) (and analogous) follows: ≤ . By summing: ∑ ≤ −

(Adil Abdullayev ,Marian Ursărescu RMM TRIANGLE MARATHON problem 214).

Hence: ≤ (and analogous);

+ + = 4 + , ∑ ≤ −

= (and analogous); = = ≥ (and analogous).

∑ ≥ + + , ≥ , ≥ ∑

By ≤ (and analogous) → ≤ (and analogous).

By summing:

≤3

4 ,1≤

14 ,

1≤

14

1+

1+

1=

14

1

+1

+1

=1;

By ≥ (andanalogous) → ≥ (and analogous) ;

Page 31: R. M. M. - 24 · 2019. 11. 29. · draga tĂtucu mariana-romania claudia nĂnuȚi-romania dan nedeianu-romania gabriela bondoc-romania ovidiu ticuȘi-romania romanian mathematical

Romanian Mathematical Society-Mehedinți Branch 2020

30 ROMANIAN MATHEMATICAL MAGAZINE NR. 24

1ℎ +

1ℎ +

1ℎ =

1;

By summing:

≥ ∑ , ≥

2 ≥ℎℎ ℎ ; ≤ 4 → ≤ 4 ; ≤ 2 ;

4 ( + + ) ≤ + + ↔ 4 (4 + ) ≤ + + ;

cos +cos + cos ≤ √ (O.Bottema-Geometric Inequalitys 1969);

3√3 ≤ ( ) →3√32 ≤ 2 ;

∑ cos ≤ ; = cos → + = cos

By summing:

cos 2 + cos 2 + cos 2 =12

+ ,

12

+≤ 2

2 ≥ + + , ≥12

2 + +

Panaitopol’s inequality :

2 ≥ ℎ → ℎ ≥ 2 ; = 2 ℎ ;

bc≥4r , bc= + → + ≥4r => ≥ (4 − )

= = ; = ;

( ) ≥ (4 − )(4 − )(4 − ), ≥ (4 − )(4 − )(4 − )

≥(4 − )(4 − )(4 − )

, 2 ≥ + +

By summing:

≥ + + + ( )( )( ) , ≥ ∑

Hence:

≥ (∑ + ( )( )( ) ; ≥ 4 → ≥

4 ≥4

; = 4 = 4 ; = ; 4 ≥

By summing:

Page 32: R. M. M. - 24 · 2019. 11. 29. · draga tĂtucu mariana-romania claudia nĂnuȚi-romania dan nedeianu-romania gabriela bondoc-romania ovidiu ticuȘi-romania romanian mathematical

Romanian Mathematical Society-Mehedinți Branch 2020

31 ROMANIAN MATHEMATICAL MAGAZINE NR. 24

34 ≥ => ≥

43 ;

= → = ∙ =( )

; ≥ ∑( )

PROPOSED PROBLEMS

5-CLASS-STANDARD

V.1. Alcătuiți o frază care să dea un sens literar expresiei: ∪ , , , , Ț ,

Proposed by Carina – Maria Viespescu– Romania

V.2. Aflați cel mai mic număr natural cu cel puțin două cifre distincte și ultima cifră nenulă, cu proprietatea că mutând ultima cifră a numărului în fața primei cifre, obținem un divizor al său. Proposed by Petre Stângescu – Romania

V.3. Find all pairs ( , ) of positive integers for which 8 = 2 + 8 + 12 + 8 + 5

Proposed by Pedro Pantoja-Brazil

V.4. Să se afle trei numere naturale consecutive < < cu proprietățile:

1) = , 2) – număr prim. Proposed by Ștefan Marica – Romania

V.5. Să se afle numărul cu proprietățile: 1) = ⋅ +

2) ( + ) = . Proposed by Ștefan Marica – Romania

V.6. a) Să se demonstreze că pentru (∀) ∈ ℕ∗, (∃) , ∈ ℕ∗ astfel încât

+ = ( + 1) + (*). b) Să se demonstreze că dacă , , ∈ ℕ∗ verifică relația (*), atunci + + 1 este pătrat perfect.

Proposed by Petre Stângescu – Romania

V.7. Find last digit of the number: Ω = . Proposed by Sorin Pîrlea – Romania

Page 33: R. M. M. - 24 · 2019. 11. 29. · draga tĂtucu mariana-romania claudia nĂnuȚi-romania dan nedeianu-romania gabriela bondoc-romania ovidiu ticuȘi-romania romanian mathematical

Romanian Mathematical Society-Mehedinți Branch 2020

32 ROMANIAN MATHEMATICAL MAGAZINE NR. 24

V.8. Determinați cel mai mic număr de 4 cifre care împărțit la 43 dă câtul egal cu restul.

Proposed by Dana Cotfasă – Romania

V.9. Let be ∈ ℕ, ≥ 3, odd. a) Find the reminder of the division of the number 4 to 5

b) Write the number 4 that the sum of four natural consecutive odd numbers.

c) Prove that the number 4 can be written as a difference between two perfect squares.

Proposed by Marin Chirciu – Romania

V.10. Let be a nonzero natural number. Solve the equation.

( + 1) + ( + 2) + ⋯+ ( + 2 + 1) = (2 + 1)

Proposed by Marin Chirciu – Romania

V.11. Prove that if 2009 can be written as a sum of squares of two nonzero natural numbers, for any ∈ ℕ∗.

Proposed by Marin Chirciu, Octavian Stroe – Romania

V.12. Compare the numbers: = 50 , = 353 , = 354 . Generalization.

Proposed by Marin Chirciu – Romania

V.13. Let be < nonzero natural numbers. Find ∈ ℕ such that the fractions:

( ) and are equivalents. Proposed by Marin Chirciu – Romania

V.14. Because 15 = 3 + 5 + 7, then number 15 can be written as a sum of at least two odd consecutive numbers. a) 2018 can be written as a sum of at least two odd consecutive numbers? If yes, then give an example of how it can be written. If no, the prove why not.

b) 2019 can be written as a sum of at least two odd consecutive numbers? If yes, the give an example of it can be written. If not, then prove why not.

Proposed by D.M. Bătineţu-Giurgiu, Neculai Stanciu-Romania

V.15. In an apartment block, there are apartments with a room and with two rooms. If 75% of the residents live in apartments with two rooms, find what is the percentage of apartments with two rooms.

Proposed by D.M. Bătineţu-Giurgiu, Neculai Stanciu-Romania

Page 34: R. M. M. - 24 · 2019. 11. 29. · draga tĂtucu mariana-romania claudia nĂnuȚi-romania dan nedeianu-romania gabriela bondoc-romania ovidiu ticuȘi-romania romanian mathematical

Romanian Mathematical Society-Mehedinți Branch 2020

33 ROMANIAN MATHEMATICAL MAGAZINE NR. 24

V.16. Find all the numbers which verify the properties:

i) { , , , , , } = {1,2,3,4,5,6}, ii) < 1, iii) < 1, iv) > 1

Proposed by Neculai Stanciu, Titu Zvonaru -Romania

V.17. a) Prove that it exists an infinity of natural nonzero consecutive numbers , , , , such that + + + + is a perfect square and + + is a perfect cube;

b) Prove that it exists an infinity of nonzero, consecutive natural numbers , , , , such that + + + + is a perfect cube and + + is a perfect square.

Proposed by D.M. Bătineţu-Giurgiu, Neculai Stanciu-Romania

V.18. a) Find all lowest nonzero, consecutive, natural numbers , , , , , , such that

+ + + + + + is a perfect square and + + + + is a perfect cube;

b) Find all the lowest nonzero, consecutive natural numbers , , , , , , such that

+ + + + + + is a perfect cube and + + + + is a perfect square.

Proposed by D.M. Bătineţu-Giurgiu, Neculai Stanciu-Romania

V.19. Prove that there is an infinity of pairs ( , ) of integers which verify the relationship

3 + 4 + 6 = 0. Proposed by D.M. Bătineţu-Giurgiu, Neculai Stanciu-Romania

V.20. Prove that there is an infinity of pairs ( , ) of integers which verify the relationship

15 + 16 + 20 = 0. Proposed by D.M. Bătineţu-Giurgiu, Neculai Stanciu-Romania

V.21. Prove that the number 100. .01 with 2012 zeros is composed.

Proposed by D.M. Bătineţu-Giurgiu, Neculai Stanciu-Romania

V.22. Find the rest of the division of the number 1 + 2 + ⋯+ 2011 to the number 2013.

Proposed by D.M. Bătineţu-Giurgiu, Neculai Stanciu-Romania

V.23. Find the rest of the division of the number 1 + 2 + ⋯+ 2013 to the number 2015.

Proposed by Titu Zvonaru-Romania

Page 35: R. M. M. - 24 · 2019. 11. 29. · draga tĂtucu mariana-romania claudia nĂnuȚi-romania dan nedeianu-romania gabriela bondoc-romania ovidiu ticuȘi-romania romanian mathematical

Romanian Mathematical Society-Mehedinți Branch 2020

34 ROMANIAN MATHEMATICAL MAGAZINE NR. 24

V.24. Find the digit such that: ( + + + ⋯+ … ): = 123456789.

Proposed by D.M. Bătineţu-Giurgiu, Neculai Stanciu-Romania

V.25. Prove that there aren’t natural numbers formed from three digits of 1 and the rest of the digits 0 which can be written as a sum of two perfect squares.

Proposed by Neculai Stanciu, Titu Zvonaru-Romania

V.26. Prove that the number 11 + 11 + 11 + 11 + 1 has an even number of divisors. Proposed by Titu Zvonaru, Neculai Stanciu -Romania

V.27. Find the numbers of three digits which divided to their reversed give the quotient 5 and the reminder a prime number. Proposed by Titu Zvonaru -Romania

All solutions for proposed problems can be finded on the http//:www.ssmrmh.ro which is the adress of Romanian Mathematical

Magazine-Interactive Journal.

6-CLASS-STANDARD

VI.1. Fie prim, ∈ ℕ∗ cu ( , ) = 1, ∈ ℕ, ≥ 2. Să se demonstreze că ecuația:

( ) − = ⋅ , nu are soluții în ℕ.

Proposed by Petre Stângescu – Romania

VI.2. Fie ∈ ℕ, ≥ 3, ∈ ℕ∗, ≤ − 1, , … , cifre cu și ≠ 0. Să se demonstreze că: … ≠ … ⋅ …

Proposed by Petre Stângescu – Romania

VI.3. Fie ∈ ℕ∗ și ecuația + = (*). Să se demonstreze că ecuația (*) are exact trei

soluții în ℕ∗ dacă și numai dacă este număr prim.

Proposed by Petre Stângescu – Romania

Page 36: R. M. M. - 24 · 2019. 11. 29. · draga tĂtucu mariana-romania claudia nĂnuȚi-romania dan nedeianu-romania gabriela bondoc-romania ovidiu ticuȘi-romania romanian mathematical

Romanian Mathematical Society-Mehedinți Branch 2020

35 ROMANIAN MATHEMATICAL MAGAZINE NR. 24

VI.4. Fie , ∈ ℕ cu 7 = 9 + 5. Demonstrați că + și nu pot fi pătrate perfecte.

Proposed by Petre Stângescu – Romania

VI.5. Să se afle suma a 20 de numere naturale distincte două câte două, știind că suma oricăror trei dintre ele este cel puțin 24, iar suma oricăror cinci dintre ele este cel mult 120.

Proposed by Petre Stângescu – Romania

VI.6. Să se arate că 2019( + ) nu este pătrat perfect, (∀) , ∈ ℕ∗

Proposed by Petre Stângescu – Romania

VI.7. Aflați număr prim, ∈ ℕ, ≥ 2, pentru care există numerele naturale , , … , distincte două câte două cu: + + ⋯+ = 2017 (*)

Proposed by Petre Stângescu – Romania

VI.8. Să se afle numărul cu proprietatea: = ⋅ + ⋅

Proposed by Ștefan Marica – Romania

VI.9. Să se afle numerele prime ; ; și știind că au loc relațiile:

1) + = + , 2) + = + = 10

Proposed by Ștefan Marica – Romania

VI.10. Find , , , ∈ ℕ; – prime number such that = + + and + + is divisible with . Proposed by Roxana Vasile, Sanda Iulia - Romania

VI.11. Aflați numărul de triunghiuri având laturile exprimate prin numere naturale iar cea mai mare latură este 11. Proposed by Doina Cristina Călina– Romania

VI.12. Prove that if and are natural numbers, prime between them, such that:

1 + + + ⋯+ = , then 2011 divides .

Proposed by D.M. Bătineţu-Giurgiu,Neculai Stanciu-Romania

VI.13. Prove that if and are natural numbers, prime between them, such that:

⋅ 1 − + − + ⋯− + + + = , then 2011 divides .

Proposed by D.M. Bătineţu-Giurgiu,Neculai Stanciu-Romania

Page 37: R. M. M. - 24 · 2019. 11. 29. · draga tĂtucu mariana-romania claudia nĂnuȚi-romania dan nedeianu-romania gabriela bondoc-romania ovidiu ticuȘi-romania romanian mathematical

Romanian Mathematical Society-Mehedinți Branch 2020

36 ROMANIAN MATHEMATICAL MAGAZINE NR. 24

VI.14. Find all the triangles that have the side’s lengths prime numbers and the square of the area natural number. Proposed by D.M. Bătineţu-Giurgiu,Neculai Stanciu-Romania

VI.15. Prove that the equation + = has an infinity of natural solutions.

Proposed by Marin Chirciu – Romania

VI.16. Prove that:

(9 + 2 ⋅ 5 + 5) ∈ ℕ, where ∈ ℕ

Proposed by Marin Chirciu – Romania

VI.17. Solve for integers:

5 − ⋅ 5 − 6 = 601

Proposed by Marin Chirciu – Romania

VI.18. Prove that there is an infinity of triplets of natural numbers ( , , ) for which the number 4 + 4 + 4 is a perfect square. Proposed by Marin Chirciu – Romania

VI.19. Let be , ∈ ℝ∗ such that + = = . Calculate ( + ) .

Proposed by Marin Chirciu – Romania

VI.20. Solve for integers: 4 − ⋅ 5 − 3 = 55.

Proposed by Marin Chirciu – Romania

VI.21. If − = − , where ≠ 0 and ≠ , calculate the product .

Proposed by Marin Chirciu – Romania

VI.22. Prove that: (5 + 2 ⋅ 3 + 1) ∈ ℕ, where ∈ ℕ.

Proposed by Marin Chirciu – Romania

VI.23. If 0 < ≤ 1, = 1,2011, prove that:

2010 + 1+

2010 + 1+ ⋯+

2010 + 1≥ 2011

Proposed by Marin Chirciu – Romania

Page 38: R. M. M. - 24 · 2019. 11. 29. · draga tĂtucu mariana-romania claudia nĂnuȚi-romania dan nedeianu-romania gabriela bondoc-romania ovidiu ticuȘi-romania romanian mathematical

Romanian Mathematical Society-Mehedinți Branch 2020

37 ROMANIAN MATHEMATICAL MAGAZINE NR. 24

VI.24. Find the numbers 0 < ≤ 1, = 1,2015, which satisfy the relationship:

2014 + 1+

2014 + 1+ ⋯+

2014 + 1= 2015

Proposed by D.M. Bătinețu-Giurgiu, Neculai Stanciu – Romania

VI.25. Prove the inequality:

12017 1 +

12 +

13 + ⋯+

12017 >

12018 1 +

12 +

13 + ⋯+

12018

Proposed by D.M. Bătinețu – Giurgiu, Neculai Stanciu – Romania

VI.26. Find all the pairs of integers ( , ) which verify the equality: 2 + 2 = .

Proposed by D.M. Bătinețu – Giurgiu, Neculai Stanciu – Romania

VI.27. Prove that there is an infinity of pairs ( , , , ) of integers which verify the relationship: (3 + 4 )(15 + 16 ) = 120

Proposed by D.M. Bătinețu-Giurgiu, Neculai Stanciu – Romania

VI.28. If , ∈ ℝ verify the relationship + + = 2, then prove that:

min(2 + 3 + 2 ) = max(7 − 8 )

Proposed D.M. Bătinețu – Giurgiu, Neculai Stanciu – Romania

All solutions for proposed problems can be finded on the http//:www.ssmrmh.ro which is the adress of Romanian Mathematical

Magazine-Interactive Journal.

7-CLASS-STANDARD

VII.1. If , , ∈ ℝ, + + = 3 then: | + ( + ) + | ≤ 4

Proposed by Daniel Sitaru – Romania

Page 39: R. M. M. - 24 · 2019. 11. 29. · draga tĂtucu mariana-romania claudia nĂnuȚi-romania dan nedeianu-romania gabriela bondoc-romania ovidiu ticuȘi-romania romanian mathematical

Romanian Mathematical Society-Mehedinți Branch 2020

38 ROMANIAN MATHEMATICAL MAGAZINE NR. 24

VII.2. Find , ∈ ℕ, – prime such that: ! = + + + ⋯+

Proposed by Seyran Ibrahimov-Azerbaijan

VII.3. Aflați ∈ ℤ astfel încât expresia = + 2 + 9 − 8 să fie cub perfect.

Proposed by Petre Stângescu – Romania

VII.4. Aflați ∈ ℕ și număr prim știind că:

= ( + ) − (2 + 1) + 1 ∈ ℚ

Proposed by Petre Stângescu – Romania

VII.5. Să se demonstreze că pentru (∀) ∈ ℕ, ≥ 2, = ≠ ℚ

Proposed by Petre Stângescu – Romania

VII.6. Dacă =√

+√

+ ⋯+√

, ∈ ℕ∗, calculați [ ]

Proposed by Petre Stângescu – Romania

VII.7. Să se demonstreze că √66 + 106 + 34 ≠ ℚ, (∀) ∈ ℕ

Proposed by Petre Stângescu – Romania

VII.8. Arătați că 7( + ) ∉ ℚ, (∀) , ∈ ℕ∗

Proposed by Petre Stângescu – Romania

VII.9. Dacă , ∈ ℕ∗ cu √2019 > , arătați că

√2019 > +73

100

Proposed by Petre Stângescu – Romania

VII.10. If , , > 0 then:

2( + ) + 3( + ) + 6( + ) ≥6 + +3 + 4 + 5

Proposed by Daniel Sitaru – Romania

Page 40: R. M. M. - 24 · 2019. 11. 29. · draga tĂtucu mariana-romania claudia nĂnuȚi-romania dan nedeianu-romania gabriela bondoc-romania ovidiu ticuȘi-romania romanian mathematical

Romanian Mathematical Society-Mehedinți Branch 2020

39 ROMANIAN MATHEMATICAL MAGAZINE NR. 24

VII.11 Să se afle numărul determinat de relația: + + = ⋅

Proposed by Ștefan Marica – Romania

VII.12. If = 3 − √3 then find Ω = .

Proposed by Tatiana Cristea – Romania

All solutions for proposed problems can be finded on the http//:www.ssmrmh.ro which is the adress of Romanian Mathematical

Magazine-Interactive Journal.

8-CLASS-STANDARD

VIII.1. I have a ≥ 4 digits positive integers which shows the following property:

9 × … = …

8 × … = …

that is when it’s multiplied by 9,8 the digits are reversed and , ∈ ℕ. Let , be general formula for above largest number respectively then for = prove that =

Proposed by Naren Bhandari-Nepal

VIII.2. Let , , > 0 and √ + √ + √ = 3√ . Prove:

2√21

√ +( , , )

−1

√ + −( , , )

≤ 3

Proposed by Nguyen Van Nho-Vietnam

VIII.3. Let > 0,∀ = 1; ( ≥ 2) and ∑ = . Prove: ∑

∑≤ √2 .

Proposed by Nguyen Van Nho-Vietnam

Page 41: R. M. M. - 24 · 2019. 11. 29. · draga tĂtucu mariana-romania claudia nĂnuȚi-romania dan nedeianu-romania gabriela bondoc-romania ovidiu ticuȘi-romania romanian mathematical

Romanian Mathematical Society-Mehedinți Branch 2020

40 ROMANIAN MATHEMATICAL MAGAZINE NR. 24

VIII.4. Let , , > 0. Prove:

++ ( + 2 )( + 2 )

++

+ ( + 2 )( + 2 )+

++ ( + 2 )( + 2 )

≥32

Proposed by Nguyen Van Nho-Vietnam

VIII.5. If , , > 0 then:

( + )( + ) ≥ 2 + 2( + )( + )( + )

Proposed by Nguyen Van Canh-Vietnam

VIII.6. If , , > 0 and = 1 then:

1− 2 + + 3 +

1− 2 + + 3 +

1− 2 + + 3 ≤ 1

Proposed by Nguyen Van Canh-Vietnam

VIII.7. If 0 < , ≤ 1 then:

+ ≥ 2 − − + 1

Proposed by Nguyen Van Canh-Vietnam

VIII.8. If , , ≥ 0 and + + = 1 then:

( + + )( + + ) ≥ + + +

Proposed by Nguyen Van Canh-Vietnam

VIII.9. Solve in ℝ:

+ +

3 + 4 + 4 ++

3 + − 8 +

4 + 4 +=

12

Proposed by Nguyen Van Canh-Vietnam

VIII.10. If , , ∈ (0,1) then: + +( )

+( )

+( )

≥ √

Proposed by D.M. Bătinețu – Giurgiu; Constantina Prunaru – Romania

Page 42: R. M. M. - 24 · 2019. 11. 29. · draga tĂtucu mariana-romania claudia nĂnuȚi-romania dan nedeianu-romania gabriela bondoc-romania ovidiu ticuȘi-romania romanian mathematical

Romanian Mathematical Society-Mehedinți Branch 2020

41 ROMANIAN MATHEMATICAL MAGAZINE NR. 24

VIII.11. If , ≥ 0; + > 0; , , > 0 then: + + ≥ ( )

Proposed by D.M. Bătinețu – Giurgiu; Oana Preda – Romania

VIII.12. If , , > 0 then:

( + ) + ( + ) + ( + ) ≥ 4 3 ( + + ) + ( − ) + ( − ) + ( − )

Proposed by D.M. Bătinețu – Giurgiu; Ramona Nălbaru – Romania

VIII.13. If , , > 0 then:

( + ) ( + )( + ) + ( + ) +

( + ) ( + )( + ) + ( + ) +

( + ) ( + )( + ) + ( + ) ≥ 2 3 ( + + )

Proposed by D.M. Bătinețu – Giurgiu; Mihaela Stăncele – Romania

VIII.14. If , , > 0, different in pairs then:

( + + )1

( − ) +1

( − ) +1

( − ) ++ +

>814

Proposed by D.M. Bătinețu – Giurgiu; Claudiu Ciulcu – Romania

VIII.15. If , , > 0; + = ; ∈ ℕ then:

√ + √√

+√ + √√

+√ − √√

+√ − √√

≥ 4

Proposed by D.M. Bătinețu – Giurgiu; Virginia Grigorescu – Romania

VIII.16. If , , > 0 then: + + + + + ≥ 2 3 ( + + )

Proposed by D.M. Bătinețu – Giurgiu; Roxana Vasile – Romania

VIII.17. If , , , , , > 0 then: + + ≥

Proposed by D.M. Bătinețu – Giurgiu; Luiza Cremeneanu – Romania

VIII.18. If , , , , , , > 0 then: ( )

+( )

+( )

≥ ( ) ( )( )

Proposed by D.M. Bătinețu – Giurgiu; Dana Cotfasă – Romania

Page 43: R. M. M. - 24 · 2019. 11. 29. · draga tĂtucu mariana-romania claudia nĂnuȚi-romania dan nedeianu-romania gabriela bondoc-romania ovidiu ticuȘi-romania romanian mathematical

Romanian Mathematical Society-Mehedinți Branch 2020

42 ROMANIAN MATHEMATICAL MAGAZINE NR. 24

VIII.19. Să se demonstre că expresia:

= 2[(2 + 1) + 2 ][(2 + 1) + (2 + 1)] + 4 + 2

nu poate fi pătrat perfect oricare ar fi , , , , , ∈ ℕ.

Proposed by Petre Stângescu – Romania

VIII.20. Aflați toate valorile lui ∈ ℕ∗ pentru care: !+

!+

!+ ⋯+

!=

Proposed by Petre Stângescu – Romania

VIII.21. If , , , , , , > 0 then:

+( + ) + ( + ) +

+( + ) + ( + ) +

+( + ) + ( + ) ≥

3+

Proposed by D.M. Bătinețu-Giurgiu, Neculai Stanciu – Romania

VIII.22. If , , , , > 0 then:

+ √+

+ √+

+ ó

+ ++

Proposed by D.M. Bătinețu-Giurgiu, Neculai Stanciu – Romania

VIII.23. If , , > 0, + + = then:

1 + + 1 + + 1 + ≥ 1

Proposed by Vasile Mircea Popa – Romania

VIII.24. If , , > 0, + + = 12 then:

14( + + ) ≤ 9 + 96

Proposed by Iuliana Trașcă – Romania

VIII.25. If , , > 0, + + = 6 then:

31 − 6 − 6+ + − 5 +

31 − 6 − 6+ + − 5 +

31 − 6 − 6+ + − 5 ≤ 7

Proposed by Iuliana Trașcă – Romania

Page 44: R. M. M. - 24 · 2019. 11. 29. · draga tĂtucu mariana-romania claudia nĂnuȚi-romania dan nedeianu-romania gabriela bondoc-romania ovidiu ticuȘi-romania romanian mathematical

Romanian Mathematical Society-Mehedinți Branch 2020

43 ROMANIAN MATHEMATICAL MAGAZINE NR. 24

VIII.26. If , , , are sides of convexe quadrilater then:

( + + − )( + + − )( + + − )( + + − )( + )( + )( + )( + ) < 1

Proposed by Daniel Sitaru – Romania

VIII.27. If , , > 0, = 1 then: ∑ + +( )

− 6 ≥ 0

Proposed by Daniel Sitaru – Romania

VIII.28. Să se afle cel puțin o pereche de numere prime ; cu proprietățile:

− = , ( + + ) − ( + + ) = , − = 1, − = 2.

Proposed by Ștefan Marica – Romania

VIII.29. Piramida patrulateră regulată are: = √7; = √ . Să se afle:

1) ( ; ) unde ∈ și = . 2) unde ⊥ ; ∈ .

Proposed by Ștefan Marica – Romania

VIII.30. Find the last 674 digists of the number: Ω = ; [∗] - great integer function.

Proposed by Nicolae Tomescu, Lucian Tuțescu – Romania

VIII.31. If , , > 0 then: max ; ; ≥ 1

Proposed by Doina Cristina Călina – Romania

All solutions for proposed problems can be finded on the http//:www.ssmrmh.ro which is the adress of Romanian Mathematical

Magazine-Interactive Journal.

9-CLASS-STANDARD

Page 45: R. M. M. - 24 · 2019. 11. 29. · draga tĂtucu mariana-romania claudia nĂnuȚi-romania dan nedeianu-romania gabriela bondoc-romania ovidiu ticuȘi-romania romanian mathematical

Romanian Mathematical Society-Mehedinți Branch 2020

44 ROMANIAN MATHEMATICAL MAGAZINE NR. 24

IX.1. In Δ the following relationship holds: sin = (1 + cos ) + 2 sin

Proposed by Mustafa Tarek-Egypt

IX.2. In Δ the following relationship holds:

√ + √ + √ ≥ 24√3( − 2 )

Proposed by Seyran Ibrahimov-Azerbaijan

IX.3. Let ∈ 0; . Prove:

(1 + sin ) + (1 + cos ) + (1 + sin ) + (1 + cos ) < 6

Proposed by Nguyen Van Nho-Vietnam

IX.4. Let , , ∈ (−1; +∞) ∧ + + = 3. Prove:

1( + + 1) +

1( + + 1) +

1( + + 1) ≥

13

Proposed by Nguyen Van Nho-Vietnam

IX.5. In Δ the following relationship holds:

cos−4 + cos

−4 + cos

−4 ≤

√ + +2

Proposed by Mustafa Tarek-Cairo-Egypt

IX.6. In Δ , – incenter the following relationship holds:

⋅ ℎ( + ℎ ) +

⋅ ℎ( + ℎ ) +

⋅ ℎ( + ℎ ) = 1

Proposed by Mustafa Tarek-Cairo-Egypt

IX.7. In Δ , – incenter the following relationship holds:

( + ℎ )⋅ ℎ +

( + ℎ )⋅ ℎ +

( + ℎ )⋅ ℎ ≥ 9

Proposed by Mustafa Tarek-Cairo-Egypt

IX.8. Solve for natural numbers:

Page 46: R. M. M. - 24 · 2019. 11. 29. · draga tĂtucu mariana-romania claudia nĂnuȚi-romania dan nedeianu-romania gabriela bondoc-romania ovidiu ticuȘi-romania romanian mathematical

Romanian Mathematical Society-Mehedinți Branch 2020

45 ROMANIAN MATHEMATICAL MAGAZINE NR. 24

= (1 − + )

= √

Proposed by Urfan Aliyev-Baku-Azerbaijan

IX.9. Let , , ≥ 0 ∧ + + = 1. Prove:

2 + 1+ + 1

( , , )

≤ 3

Proposed by Nguyen Van Nho- Vietnam

IX.10. If , > 0, ≥ then:

sin+

sin>

1

cos + sin

Proposed by Daniel Sitaru – Romania

IX.11. In Δ the following relationship holds:

ℎ + ℎℎ =

− ℎ ℎ ℎ

Proposed by Bogdan Fustei-Romania

IX.12. In Δ the following relationship holds:

23( + + )

≤ 2

Proposed by Seyran Ibrahimov-Azerbaijan

IX.13. In Δ the following relationship holds:

sin cot 2 + sin cot 2 + sin cot 2 ≤ 3 −12

Proposed by Marian Ursărescu-Romania

IX.14. In Δ , – incentre the following relationship holds:

Page 47: R. M. M. - 24 · 2019. 11. 29. · draga tĂtucu mariana-romania claudia nĂnuȚi-romania dan nedeianu-romania gabriela bondoc-romania ovidiu ticuȘi-romania romanian mathematical

Romanian Mathematical Society-Mehedinți Branch 2020

46 ROMANIAN MATHEMATICAL MAGAZINE NR. 24

ℎ + ℎ + ℎ ≤3√3

2

Proposed by Bogdan Fustei-Romania

IX.15. Solve in ℝ:

√ − + 1 + √ − 3 + 1 + √ + + 1 + √ + 3 + 1+ 2 = 2

Proposed by Nguyen Van Nho-Vietnam

IX.16. In Δ , – incentre the following relationship holds:

+ℎ +

+ℎ +

+ℎ ≤

+ +

Proposed by Bogdan Fustei-Romania

IX.17.

= 6 tan327 + 4 sin

227 −

1496646325× 10

This form is accurate up to 10th decimal place. More, interestingly with further simplification we can obtain

tan327 + 4 sin

227 −

49888215 × 10 = 6

Proposed by Naren Bhandari-Nepal

IX.18. Find all functions :ℝ → ℝ satisfying:

( + ) ≥ ( + 1) ( ),∀ , ∈ ℝ, 1 ≤ ∈ ℕ

Proposed by Nguyen Van Canh-Vietnam

IX.19. In any triangle holds:

+ + 42 ≤ + + ≤ 4 +

Proposed by Nguyen Van Canh-Vietnam

Page 48: R. M. M. - 24 · 2019. 11. 29. · draga tĂtucu mariana-romania claudia nĂnuȚi-romania dan nedeianu-romania gabriela bondoc-romania ovidiu ticuȘi-romania romanian mathematical

Romanian Mathematical Society-Mehedinți Branch 2020

47 ROMANIAN MATHEMATICAL MAGAZINE NR. 24

IX.20. In all triangle holds: 1. max ∑ ;∑ ≤ 4 + , 2. min ∑ ;∑ ≥

Proposed by Nguyen Van Canh-Vietnam

IX.21. If , , > 0 then in Δ the following relationship holds:

2 sin + 2 sin + 2 sin+ + ≤

+ +

Proposed by Nguyen Van Canh-Vietnam

IX.22. Find all functions :ℝ → ℝ:

= ( ) ( ) ,∀ , ∈ ℝ, , ∈ ℕ − {0}− fixed

Proposed by Nguyen Van Canh-Vietnam

IX.23. Let be Δ and , , the middles of , , arcs (made with the circumcenter). Prove that:

⋅ ⋅⋅ ⋅ ≤ cos

−2 cos

−2 cos

−2

Proposed by Marian Ursărescu – Romania

IX.24. Prove that in any acute-angled triangle the following inequality holds:

cos sin(sin ) + cos (sin(sin ) + cos ⋅ sin(sin )) ≤32 sin

√34

Proposed by Marian Ursărescu – Romania

IX.25. Let be Δ , , and the internal bisectors. Prove that:

⋅ + ⋅ + ⋅ ≥ 18

Proposed by Marian Ursărescu – Romania

IX.26. Find the functions :ℕ∗ → ℕ∗ having the property:

(1)− (2) + (3) … + (−1) ( ) =

Page 49: R. M. M. - 24 · 2019. 11. 29. · draga tĂtucu mariana-romania claudia nĂnuȚi-romania dan nedeianu-romania gabriela bondoc-romania ovidiu ticuȘi-romania romanian mathematical

Romanian Mathematical Society-Mehedinți Branch 2020

48 ROMANIAN MATHEMATICAL MAGAZINE NR. 24

= ( + − 1) (1) − (2) + ⋯+ (−1) ( ) ;∀ ≥ 1

Proposed by Marian Ursărescu – Romania

IX.27. Find all :ℝ → ℝ satisfying: ( ) = 4 ( )− 3 ,∀ ∈ ℝ

Proposed by Nguyen Van Canh-Vietnam

IX.28. In Δ , ≥ ≥ , + ≥ 3 . Prove that: 4 − 9 ≥ 0.

Proposed by Nguyen Van Canh-Vietnam

IX.29. Prove that in any Δ holds the following inequality: + + 6∑ ≥ 16

Proposed by Marian Ursărescu – Romania

IX.30. Let be Δ and , , the interior bisectors (concurrent in ). Prove that:

⋅ + ⋅ + ⋅ ≥643 ⋅

Proposed by Marian Ursărescu – Romania

IX.31. Prove that in any acute-angled Δ the following inequality holds:

+ + ≥ 4 ( + )

Proposed by Marian Ursărescu – Romania

IX.32. Prove that in any Δ the following inequality holds:

8 cos 2 − cos 2 ≤ 15

Proposed by Marian Ursărescu – Romania

IX.33. Prove that in any Δ the following inequality holds:

(ℎ + ℎ )(ℎ + ℎ )ℎ ℎ ≥ 768 ⋅

Proposed by Marian Ursărescu – Romania

IX.34. Prove that in any acute-angled triangle the following inequality holds:

Page 50: R. M. M. - 24 · 2019. 11. 29. · draga tĂtucu mariana-romania claudia nĂnuȚi-romania dan nedeianu-romania gabriela bondoc-romania ovidiu ticuȘi-romania romanian mathematical

Romanian Mathematical Society-Mehedinți Branch 2020

49 ROMANIAN MATHEMATICAL MAGAZINE NR. 24

ℎ+ℎ

+ℎ

≥12

Proposed by Marian Ursărescu – Romania

IX.35. In Δ the following relationship holds:

( + ) sinℎ + ℎ +

( + ) sinℎ + ℎ +

( + ) sinℎ + ℎ = 3

Proposed by Bogdan Fustei-Romania

IX.36. In Δ the following relationship holds: + + = 3

Proposed by Bogdan Fustei-Romania

IX.37. In Δ the following relationship holds:

6 + ℎ + ℎ + ℎ ≥ 2( + + )

Proposed by Bogdan Fustei-Romania

IX.38. In Δ the following relationship holds:

+ℎ + ℎ

≥ 6 sin 2

Proposed by Bogdan Fustei-Romania

IX.39. In Δ , – incenter the following relationship holds:

ℎ + ℎ ≥+ +

4 ≥ ℎ + ℎ

Proposed by Bogdan Fustei-Romania

IX.40. If in Δ , – incentre then:

1( + + ) ≥

1( + ) +

1( + ) +

1( + )

Proposed by Bogdan Fustei-Romania

Page 51: R. M. M. - 24 · 2019. 11. 29. · draga tĂtucu mariana-romania claudia nĂnuȚi-romania dan nedeianu-romania gabriela bondoc-romania ovidiu ticuȘi-romania romanian mathematical

Romanian Mathematical Society-Mehedinți Branch 2020

50 ROMANIAN MATHEMATICAL MAGAZINE NR. 24

IX.41. If , , > 0 then in Δ the following relationship holds:

+ + ≥ 6√3 ; ( – area)

Proposed by D.M. Bătinețu-Giurgiu, Gabriel Tică – Romania

IX.42. If , , > 0 then in Δ the following relationship holds:

+cos 2 +

+cos 2 +

+cos 2 ≥

4 − 94

Proposed by D.M. Bătinețu-Giurgiu, Lavinia Trinu – Romania

IX.43. If , , > 0 then in Δ the following relationship holds:

+ + ≥ 6√3 ; ( – area)

Proposed by D.M. Bătinețu-Giurgiu, Alexandrina Năstase – Romania

IX.44. If , > 0 then in Δ the following relationship holds:

( + + )( )

+( )

+( )

≥( )

; ( – area)

Proposed by D.M. Bătinețu-Giurgiu, Cătălina Pană – Romania

IX.45. In Δ the following relationship holds: + + ≥ √

Proposed by D.M. Bătinețu-Giurgiu, Neculai Stanciu – Romania

IX.46. If , > 0 then in Δ the following relationship holds:

( + ) + ( + ) + ( + ) ≥48

+

Proposed by D.M. Bătinețu-Giurgiu, Neculai Stanciu – Romania

IX.47. If ≥ 0 then in Δ the following relationship holds:

( ) + ( ) + ( ) ≥ 3 + + 4

Proposed by D.M. Bătinețu-Giurgiu, Claudia Nănuți – Romania

Page 52: R. M. M. - 24 · 2019. 11. 29. · draga tĂtucu mariana-romania claudia nĂnuȚi-romania dan nedeianu-romania gabriela bondoc-romania ovidiu ticuȘi-romania romanian mathematical

Romanian Mathematical Society-Mehedinți Branch 2020

51 ROMANIAN MATHEMATICAL MAGAZINE NR. 24

IX.48. If , , , > 0 then in Δ the following relationship holds:

+ ( − − 4 )+

++ ( − − 4 )

++

+ ( − − 4 )+

≥(2 + 3 )

+

Proposed by D.M. Bătinețu-Giurgiu – Romania, Martin Lukarevski – Macedonia

IX.49. If , , > 0 then:

3( + + )+ + ≤ 2( + + )

14 + + +

1+ 4 + +

1+ + 4 ≤ 9

Proposed by D.M. Bătinețu-Giurgiu, Daniel Sitaru – Romania

IX.50. If : (0,∞) → (0,∞) is such that: ( ) + ( ) ≤ 2 , (∀) , > 0 then

( ) + ( ) + ( ) ≤ 33

+ + ; (∀) , , > 0

Proposed by D.M. Bătinețu-Giurgiu, Daniel Sitaru – Romania

IX.51. In Δ the following relationship holds: + + ≥

Proposed by D.M. Bătinețu-Giurgiu – Romania

IX.52. Prove that:

cos317 + cos

517 − cos

617 + cos

717 =

1 + √174

Proposed by Vasile Mircea Popa – Romania

IX.53. Let Δ be the circumcevian triangle of incenter in Δ .

If { } = ∩ , { } = ∩ , { } = ∩ then:

+ + ≤(4 + )

3

Proposed by Marian Ursărescu – Romania

IX.54. Solve for ∈ 0, : sin + cos + tan + cot + (sec + csc ) = 2 1 + √2

Proposed by Daniel Sitaru – Romania

Page 53: R. M. M. - 24 · 2019. 11. 29. · draga tĂtucu mariana-romania claudia nĂnuȚi-romania dan nedeianu-romania gabriela bondoc-romania ovidiu ticuȘi-romania romanian mathematical

Romanian Mathematical Society-Mehedinți Branch 2020

52 ROMANIAN MATHEMATICAL MAGAZINE NR. 24

IX.55. In Δ ; cos = ; cos = . Find cos 3 .

Proposed by Ileana Duma – Romania

IX.56. Exists :ℝ → ℝ; ( ) = + + ; , , ∈ ℝ; ≠ 0 such that:

(2 ) = ( + + 1), (∀) ∈ ℝ ?

Proposed by Camelia Dană, Cristian Moanță – Romania

All solutions for proposed problems can be finded on the http//:www.ssmrmh.ro which is the adress of Romanian Mathematical

Magazine-Interactive Journal.

10-CLASS-STANDARD

X.1. In Δ , , , – circumradii of Δ ,Δ ,Δ

, , – excenters, – Bevan’s point. Prove that:

+ + ≥92

Proposed by Mehmet Sahin-Turkey

X.2. In Δ , – incentre the following relationship holds:

ℎ − 2 ≥ 1 + 8+ + + + +

+ + + ℎ + ℎ + ℎ − 3

Proposed by Bogdan Fustei – Romania

X.3. In Δ , , , – excenters, = (40) – Bevan’s point

, , – circumradii in Δ ,Δ ,Δ . Prove that:

Page 54: R. M. M. - 24 · 2019. 11. 29. · draga tĂtucu mariana-romania claudia nĂnuȚi-romania dan nedeianu-romania gabriela bondoc-romania ovidiu ticuȘi-romania romanian mathematical

Romanian Mathematical Society-Mehedinți Branch 2020

53 ROMANIAN MATHEMATICAL MAGAZINE NR. 24

ℎ+ℎ

+ℎ

= 2( + + )

Proposed by Mehmet Sahin-Turkey

X.4. In Δ the following relationship holds:

≥32 ⋅min ( − ) , ( − ) , ( − )

Proposed by Marian Ursărescu – Romania

X.5. In Δ the following relationship holds:

+ + ≥2

− 1

Proposed by Adil Abdullayev-Baku-Azerbaijan

X.6. In Δ the following relationship holds:

+ + ≤+ +

3

Proposed by Adil Abdullayev-Baku-Azerbaijan

X.7. In Δ , , , – circumradii of Δ ,Δ ,Δ , , , – excenters, – Bevan’ s point. Prove that:

1=

2 −2 , =

4, + + =

[ ]− 2[ ]2

Proposed by Mehmet Sahin-Turkey

X.8. Solve for natural numbers:

⎩⎨

⎧ = −

1( ) =

Proposed by Urfan Aliyev-Baku-Azerbaijan

X.9. Solve for real numbers:

Page 55: R. M. M. - 24 · 2019. 11. 29. · draga tĂtucu mariana-romania claudia nĂnuȚi-romania dan nedeianu-romania gabriela bondoc-romania ovidiu ticuȘi-romania romanian mathematical

Romanian Mathematical Society-Mehedinți Branch 2020

54 ROMANIAN MATHEMATICAL MAGAZINE NR. 24

+ =( ) − ( + + )

( + + )( + ) =

Proposed by Urfan Aliyev-Baku-Azerbaijan

X.10. In Δ , – Lemoine’s point. Prove that:

+ + ≤√33 sin + sin + sin

Proposed by Mustafa Tarek-Cairo-Egypt

X.11. In Δ , – Mittenpunkt, , , – excenters. Prove that:

[ ]

cos=

[ ]

cos=

[ ]

cos

Proposed by Mustafa Tarek-Cairo-Egypt

X.12. Solve for natural numbers:

( + 3) = (5 − − )

( + ) = (8 + 3 )

Proposed by Urfan Aliyev-Baku-Azerbaijan

X.13. Prove that if – is the circumcenter of external triangle of Δ (Bevan’s point) then:

+ + = 12 − − − 4

+ + = 4 ([ ] − 3{ })

Proposed by Mehmet Sahin-Turkey

X.14. In Δ , , , – circumradii of Δ ,Δ ,Δ , , , – excenters, – Bevan’s point. Prove that:

1=

2 −2 , =

4, + + =

[ ] − 2[ ]2

Proposed by Mehmet Sahin-Turkey

X.15. Prove that exact value in radical form of

Page 56: R. M. M. - 24 · 2019. 11. 29. · draga tĂtucu mariana-romania claudia nĂnuȚi-romania dan nedeianu-romania gabriela bondoc-romania ovidiu ticuȘi-romania romanian mathematical

Romanian Mathematical Society-Mehedinți Branch 2020

55 ROMANIAN MATHEMATICAL MAGAZINE NR. 24

sin 3∘ =14 8 − 10 − 2√5 − √3 − √15 =

14 8 − √10 √10 − √2 − √3 − √15

In radian form

sin 3∘ = sin 60 =12 1 − sin

815 = − cos

3160 ⋅ sin 60

Proposed by Naren Bhandari-Nepal

X.16. In any triangle , show that:

(4 + )(2 − ) ≥ max{ℎ ;ℎ ;ℎ }

Proposed by Nguyen Van Canh-Vietnam

X.17. Find all functions :ℝ → ℝ such that:

( + ) ≥ ( ) ( ) ≥ 2018

Proposed by Nguyen Van Canh-Vietnam

X.18. In Δ the following relationship holds:

+ + ≥94

Proposed by Nguyen Van Canh-Vietnam

X.19. Let be Δ and , and the internal bisectors of Δ . Find:

≤ + +32

Proposed by Marian Ursărescu – Romania

X.20. Let be Δ , the intersection point of the symmedians and , , the intersection points of the cevians , , and with the circumcenter of Δ . Prove that:

+ + ≥( + + )

+ +

Proposed by Marian Ursărescu – Romania

Page 57: R. M. M. - 24 · 2019. 11. 29. · draga tĂtucu mariana-romania claudia nĂnuȚi-romania dan nedeianu-romania gabriela bondoc-romania ovidiu ticuȘi-romania romanian mathematical

Romanian Mathematical Society-Mehedinți Branch 2020

56 ROMANIAN MATHEMATICAL MAGAZINE NR. 24

X.21. Let be Δ and the intersection point of the symmedians. Prove that:

+ + ≥( + + )+ +

Proposed by Marian Ursărescu – Romania

X.22. If , , > 0 then in Δ the following relationship holds:

sin + sin + sin ≤( + + )

4

Proposed by Nguyen Van Canh-Vietnam

X.23. Prove that in any acute-angled triangle the following inequality holds:

min( cot , cot , cot ) ≤ 2 ≤ max( cot , cot , cot )

Proposed by Marian Ursărescu – Romania

X.24. Prove that in any Δ the following inequality holds:

cos+

cos+

cos≤

332 ⋅

Proposed by Marian Ursărescu – Romania

X.25. In Δ , – incenter the following relationship holds:

+ + ≥ + 3 2 − √3

Proposed by Bogdan Fustei-Romania

X.26. In Δ , – incentre, , , – circumradii in Δ ,Δ ,Δ the following relationship holds:

ℎ + ℎ + ℎ ≤3√3

2

Proposed by Bogdan Fustei-Romania

X.27. In Δ the following relationship holds: + + ≤ + +

Proposed by Bogdan Fustei-Romania

Page 58: R. M. M. - 24 · 2019. 11. 29. · draga tĂtucu mariana-romania claudia nĂnuȚi-romania dan nedeianu-romania gabriela bondoc-romania ovidiu ticuȘi-romania romanian mathematical

Romanian Mathematical Society-Mehedinți Branch 2020

57 ROMANIAN MATHEMATICAL MAGAZINE NR. 24

X.28. In Δ the following relationship holds:

ℎ + ℎ + ℎ +2

+2

+2

≤+ +

Proposed by Bogdan Fustei-Romania

X.29. In Δ the following relationship holds:

+ + ≥ 2 ⋅+ℎ

Proposed by Bogdan Fustei-Romania

X.30. In Δ the following relationship holds:

cot+ +

cot+ +

cot+ = 3

Proposed by Bogdan Fustei-Romania

X.31. In Δ the following relationship holds:

ℎ − 2 + ℎ − 2 + ℎ − 2 ≥4 +

Proposed by Bogdan Fustei-Romania

X.32. In Δ the following relationship holds:

2+

2+

2≤

+− +

+− +

+−

Proposed by Bogdan Fustei-Romania

X.33. In Δ the following relationship holds:

ℎ + ℎℎ +

ℎ + ℎℎ +

ℎ + ℎℎ =

2⋅

ℎ⋅

Proposed by Bogdan Fustei-Romania

X.34. In Δ the following relationship holds:

Page 59: R. M. M. - 24 · 2019. 11. 29. · draga tĂtucu mariana-romania claudia nĂnuȚi-romania dan nedeianu-romania gabriela bondoc-romania ovidiu ticuȘi-romania romanian mathematical

Romanian Mathematical Society-Mehedinți Branch 2020

58 ROMANIAN MATHEMATICAL MAGAZINE NR. 24

−( − )( − ) +

−( − )( − ) +

−( − )( − ) =

2 −√

Proposed by Bogdan Fustei-Romania

X.35. In Δ the following relationship holds:

+ +≥

++

++

+

Proposed by Bogdan Fustei-Romania

X.36. In Δ the following relationship holds:

13 2 ℎ ℎ + ℎ ℎ + ℎ ℎ ≥

+ ++ +

Proposed by Bogdan Fustei-Romania

X.37. Let Δ be the circumcevian triangle of – incenter in Δ . Prove that:

+ + ≥ +52

Proposed by Marian Ursărescu – Romania

X.38. If ∈ ℕ; , , > 0 then in Δ the following relationship holds:

+ 3 ⋅ ∑ ( ) ≥ 4( + 1) + + ; ( – area)

Proposed by D.M. Bătinețu-Giurgiu, Constantin Basarab – Romania

X.39. If ∈ ℕ then in Δ the following relationship holds:

+13 cot 2 ≥ 6 ( + 1)

Proposed by D.M. Bătinețu-Giurgiu, Marian Voinea – Romania

X.40. In Δ the following relationshp holds: ∑( )( )

≥ 36

Proposed by D.M. Bătinețu-Giurgiu, Alecu Orlando – Romania

Page 60: R. M. M. - 24 · 2019. 11. 29. · draga tĂtucu mariana-romania claudia nĂnuȚi-romania dan nedeianu-romania gabriela bondoc-romania ovidiu ticuȘi-romania romanian mathematical

Romanian Mathematical Society-Mehedinți Branch 2020

59 ROMANIAN MATHEMATICAL MAGAZINE NR. 24

X.41. In Δ the following relationship holds:

+2

(4 + ) − 2 ≤ (4 + )

Proposed by D.M. Bătinețu-Giurgiu, Camelia Dană – Romania

X.42. In Δ the following relationship holds:

cot 2 + cot 2 + cot 2 ≥ 8(4 + ) ≥ 8√3

Proposed by D.M. Bătinețu-Giurgiu, Alina Țigae – Romania

X.43. If , , > 0 then in Δ the following relationship holds:

( )+

( )+

( )≥ √ ; ( – area)

Proposed by D.M. Bătinețu-Giurgiu, Anicuța Betiu – Romania

X.44. If , , > 0 then in Δ the following relationship holds:

( + ) + + ( + ) + ( + ) + ≥ 48

Proposed by D.M. Bătinețu-Giurgiu, Neculai Stanciu – Romania

X.45. If ∈ ℕ then in Δ the following relationship holds:

1( + ) +

1( + ) +

1( + ) ≥

9( + 1)4(( ) + ( ) + ( ) ) + 12

Proposed by D.M. Bătinețu-Giurgiu, Claudia Nănuți – Romania

X.46. If , , , , , > 0 then:

( + ( + ) )( + ) +

( + ( + ) )( + ) +

( + ( + ) )( + ) ≥

≥32

( + + ) − 2( + + )

Proposed by D.M. Bătinețu-Giurgiu – Romania, Martin Lukarevski – Macedonia

Page 61: R. M. M. - 24 · 2019. 11. 29. · draga tĂtucu mariana-romania claudia nĂnuȚi-romania dan nedeianu-romania gabriela bondoc-romania ovidiu ticuȘi-romania romanian mathematical

Romanian Mathematical Society-Mehedinți Branch 2020

60 ROMANIAN MATHEMATICAL MAGAZINE NR. 24

X.47. In Δ the following relationship holds:

tan + tan tan + tan

1 + tan 1 + tan⋅

1

tan + tan≥

94

Proposed by D.M. Bătinețu-Giurgiu, Daniel Sitaru – Romania

X.48. Prove that:

cos419 + cos

619 − cos

919 =

√193 cos

13 cos

72√19

−16

Proposed by Vasile Mircea Popa – Romania

X.49. In acute Δ the following relationship holds:

√tan + √tan + √tan − √cot − √cot − √cot ≥ 3√3 −3√3

Proposed by Vasile Mircea Popa – Romania

X.50. Prove that for any acute or right triangle the following inequalities holds:

3 √3 − 1√2

≤ √sin + √sin + √sin − √cos − √cos − √cos ≤ 1

Proposed by Vasile Mircea Popa – Romania

X.51. In acute Δ the following relationship holds:

cos + cos + cos +32 ≤ 2 sin 2 + sin 2 + sin 2

Proposed by Vasile Mircea Popa – Romania

X.52. In Δ the following relationship holds:

6 sin + 3 sin 2 + 2 sin 3 > 0

Proposed by Daniel Sitaru – Romania

X.53. Prove that for all , , ∈ (0,∞) and any natural number ≥ 3 , we have:

Page 62: R. M. M. - 24 · 2019. 11. 29. · draga tĂtucu mariana-romania claudia nĂnuȚi-romania dan nedeianu-romania gabriela bondoc-romania ovidiu ticuȘi-romania romanian mathematical

Romanian Mathematical Society-Mehedinți Branch 2020

61 ROMANIAN MATHEMATICAL MAGAZINE NR. 24

1√ + + ≥

3√( + + ) + − 1

Proposed by Daniel Sitaru – Romania

X.54. In Δ the following relationship holds:

+ + ≥ ℎ + √ ℎ + ℎ , , , ≥ 0

Proposed by Daniel Sitaru – Romania

X.55. If , , ∈ ℂ − {0} – different in pairs, | | = | | = | | = 1, ( ), ( ), ( )

( )| |

+ ( )| |

+ ( )| |

+ 3√3 = 0 then: = =

Proposed by Marian Ursărescu – Romania

X.56. In Δ let Ω be area of intouch triangle, Ω area of circumcevian triangle of incenter. Prove that: Ω = Ω

Proposed by Marian Ursărescu – Romania

X.57. In acute Δ the following relationship holds:

maxsin 2

cos( − ) ,sin 2

cos( − ) ,sin 2

cos( − ) ≥ 2√3

Proposed by Marian Ursărescu – Romania

X.58. Let Δ be the circumcevian triangle of othocenter in acute Δ . Prove that:

⋅ + ⋅ + ⋅ ≥ 21

+1

Proposed by Marian Ursărescu – Romania

X.59. ∈ ℝ, ≠ 1, ( + + ), ( + + ), ( + + )

( ), ( ), ( ), , , ∈ ℂ. Prove that: = = ⇒ = =

Proposed by Marian Ursărescu – Romania

X.60. If , , ∈ ℂ − {0} – different in pairs, | | = | | = | | = 1, ( ), ( ), ( )

Page 63: R. M. M. - 24 · 2019. 11. 29. · draga tĂtucu mariana-romania claudia nĂnuȚi-romania dan nedeianu-romania gabriela bondoc-romania ovidiu ticuȘi-romania romanian mathematical

Romanian Mathematical Society-Mehedinți Branch 2020

62 ROMANIAN MATHEMATICAL MAGAZINE NR. 24

( )| |

+ ( )| |

+ ( )| |

= 3 then: = = .

Proposed by Marian Ursărescu – Romania

X.61 … – regular polygon, ∈ ℕ, ≥ 3, ( ), ( ), … , ( ), ∈ ℂ, ∈ 1,

If (0) – centre of polygon and exists , ∈ 1, , ≠ such that ⋅ + ⋅ = 0 then:

is divisible with 4.

Proposed by Marian Ursărescu – Romania

X.62. Prove that = cos , = cos , = cos are roots of equation:

4 − 4 ( + ) + ( + − 4 ) + (2 + ) − = 0

Proposed by Marian Ursărescu – Romania

X.63. Let , , be centers of circles tangent each one at two sides of triangle and extangent to circumcircle. If – incenter then: ⋅ + ⋅ + ⋅ ≥ 36

Proposed by Marian Ursărescu – Romania

X.64. BĂTINEȚU’S INEQUALITY – 1

If , , > 0 then in Δ the following relationship holds:

+⋅ +

+⋅ +

+⋅ ≥ 8√3 ⋅

Proposed by D.M. Bătinețu-Giurgiu – Romania

X.65. BĂTINEȚU’S INEQUALITY – 2

If , , > 0 then in Δ the following relationship holds:

+⋅ +

+⋅ +

+⋅ ≥ 32 ⋅

Proposed by D.M. Bătinețu-Giurgiu – Romania

X.66. Let be a ring with identity. For each ∈ we define: ≔ { ∈ : = 1}

Show that if ∈ and | | ≥ 2 then the function : → defined by

( ) = + − 1 is injective but not surjective. Proposed by Oleg Țurcan – Portugal

Page 64: R. M. M. - 24 · 2019. 11. 29. · draga tĂtucu mariana-romania claudia nĂnuȚi-romania dan nedeianu-romania gabriela bondoc-romania ovidiu ticuȘi-romania romanian mathematical

Romanian Mathematical Society-Mehedinți Branch 2020

63 ROMANIAN MATHEMATICAL MAGAZINE NR. 24

X.67. If , , > 0 then:

√+ + ≤ + +

Proposed by Daniel Sitaru – Romania

X.68. If , , , > 0, + + + = 1 then:

1−

1−

1−

1− ≥

25516

Proposed by Daniel Sitaru – Romania

X.69. If , , , are sides in a cyclic quadrilateral, , , , – exradii, – semiperimeter then:

+ + + ≥32

Proposed by Daniel Sitaru – Romania

X.70. If , , , are sides in a cyclic quadrilateral, , , , – exradii, – semiperimeter then:

+ + + ≥2

Proposed by Daniel Sitaru – Romania

X.71. In Δ the following relationship holds:

+ + ≤ + +

Proposed by Daniel Sitaru – Romania

X.72. If , , > 0, + + √ = 3 then:

− √ + − + − √ + 2 − 1 ≥ 0

Proposed by Daniel Sitaru – Romania

Page 65: R. M. M. - 24 · 2019. 11. 29. · draga tĂtucu mariana-romania claudia nĂnuȚi-romania dan nedeianu-romania gabriela bondoc-romania ovidiu ticuȘi-romania romanian mathematical

Romanian Mathematical Society-Mehedinți Branch 2020

64 ROMANIAN MATHEMATICAL MAGAZINE NR. 24

X.73. Let be :ℝ → ℝ; ( ) = sin + √ + 4. If (lg(log 10)) = 5 then prove that (lg(lg 3)) = 3

Proposed by Simona Radu, Simona Miu– Romania

X.74. If , , > 0 then in Δ the following relationship holds:

+ + ( + + ) ≥ 96 ; ( – area)

Proposed by D.M. Bătinețu – Giurgiu; Dan Nănuți – Romania

X.75. If , , ≥ 0 then in Δ the following relationship holds:

+ + ≥ ; ( – area)

Proposed by D.M. Bătinețu – Giurgiu; Neculai Stanciu – Romania

All solutions for proposed problems can be finded on the http//:www.ssmrmh.ro which is the adress of Romanian Mathematical

Magazine-Interactive Journal.

11-CLASS-STANDARD

XI.1. Solve for real numbers:

2 + 3 + 4 + 192 + 648 + 1536 = 48 + 72 + 96 + 3 ⋅ 24

Proposed by Rovsen Pirguliyev-Azerbaijan

XI.2. Find:

Ω = lim→

1− 90 +1

,Ω = lim→

4 − 3 +1

( + )

Page 66: R. M. M. - 24 · 2019. 11. 29. · draga tĂtucu mariana-romania claudia nĂnuȚi-romania dan nedeianu-romania gabriela bondoc-romania ovidiu ticuȘi-romania romanian mathematical

Romanian Mathematical Society-Mehedinți Branch 2020

65 ROMANIAN MATHEMATICAL MAGAZINE NR. 24

Ω = lim→

5− 4 log 2 − 6 +1

(2 + 1) ,Ω = lim→

1 − 8 +1

(2 − 1)

Proposed by Daniel Sitaru – Romania

XI.3. Find:

Ω = lim→

1− log(1 + ) , =

25

2 −

Proposed by Daniel Sitaru – Romania

XI.4 Prove without softs: 2018√ < √

Proposed by Rovsen Pirguliyev-Azerbaijan

XI.5. Find all ROLLE functions : [0,1] → ℝ such that:

(0) = (1) =20192018

2017 ( ) + 2018 ( ) ≤ 2019,∀ ∈ (0,1)

Proposed by Nguyen Van Canh-Vietnam

XI.6. Let , , denote the angle of triangle . Find the maximum value of:

= cos cos + cos

Proposed by Nguyen Van Canh-Vietnam

XI.7. Find:

lim→

ln(1 + sinh ) − ln (1 + sinh ), ∈ ℕ∗, ≥ 2

Proposed by Marian Ursărescu – Romania

XI.8. Find:

lim → + + ⋯+ − , where > 1, ∈ ℕ∗

Proposed by Marian Ursărescu – Romania

Page 67: R. M. M. - 24 · 2019. 11. 29. · draga tĂtucu mariana-romania claudia nĂnuȚi-romania dan nedeianu-romania gabriela bondoc-romania ovidiu ticuȘi-romania romanian mathematical

Romanian Mathematical Society-Mehedinți Branch 2020

66 ROMANIAN MATHEMATICAL MAGAZINE NR. 24

XI.9. Find:

lim→

1+ 1 + + 1

Proposed by Marian Ursărescu – Romania

XI.10. Let be , ∈ (ℝ) such that = , det = , > 0 and det( + ) = 0. Find: det( − + )

Proposed by Marian Ursărescu – Romania

XI.11. Let be = ∫ ( ) ( ) , ∈ ℕ∗ , ≥ 2. Find such that ∈ ℚ.

Proposed by Marian Ursărescu – Romania

XI.12. Find the continuous functions :ℝ → (0, +∞) having the property:

( ) ⋅ (2 ) ⋅ (4 ) = 2 ,∀ ∈ ℝ

Proposed by Marian Ursărescu – Romania

XI.13. Let be = − + … and = − + … Find:

lim→

+ +3

Proposed by Marian Ursărescu – Romania

XI.14. Let be , ∈ (ℝ) such that 5 − √5( + ) = 3 + √5( − ) and

− is invertible. Prove that ⋮ 5.

Proposed by Marian Ursărescu – Romania

XI.15. In Δ the following relationship holds:

cos( − ) + cos( − ) + cos( − ) ≤12

++

++

+

Proposed by Nguyen Van Canh-Vietnam

XI.16. Let , , , > 0. Find the maximum value of:

Page 68: R. M. M. - 24 · 2019. 11. 29. · draga tĂtucu mariana-romania claudia nĂnuȚi-romania dan nedeianu-romania gabriela bondoc-romania ovidiu ticuȘi-romania romanian mathematical

Romanian Mathematical Society-Mehedinți Branch 2020

67 ROMANIAN MATHEMATICAL MAGAZINE NR. 24

= ( + )( + ) + ( + )( + ) + ( + )( + ) + ( + )( + )

Proposed by Nguyen Van Canh-Vietnam

XI.17. Find all functions :ℝ → ℝ continuous in = 0 such that:

(2018 ) = (2019 ) +

Proposed by Nguyen Van Canh-Vietnam

XI.18. Let be , , ∈ ℝ, ≠ 0. If det( + + ) ≥ 0,∀ ∈ (ℝ), then:

− 4 ≤ 0. Proposed by Marian Ursărescu – Romania

XI.19. Does it exist , ∈ (ℚ) such that: ( − ) + = ?

Proposed by Marian Ursărescu – Romania

XI.20. Find the continuous functions : (0, +∞) → ℝ having the property:

=( ) + ( )

2 ,∀ , > 0

Proposed by Marian Ursărescu – Romania

XI.21. Find: lim → − ( )

( ), ∈ ℕ∗

Proposed by Marian Ursărescu – Romania

XI.22. Let be the sequences > 0, = 1 + − 1,∀ ∈ ℕ and > 0,

= + , ∈ ℕ, ≥ 2. Find: lim → ⋅

Proposed by Marian Ursărescu – Romania

XI.23. Solve the equation: + (2 − 1) = 21 1 00 1 10 0 1

, where ∈ (ℝ) and

∈ ℕ, ≥ 2

Proposed by Marian Ursărescu – Romania

Page 69: R. M. M. - 24 · 2019. 11. 29. · draga tĂtucu mariana-romania claudia nĂnuȚi-romania dan nedeianu-romania gabriela bondoc-romania ovidiu ticuȘi-romania romanian mathematical

Romanian Mathematical Society-Mehedinți Branch 2020

68 ROMANIAN MATHEMATICAL MAGAZINE NR. 24

XI.24. Prove that in any Δ the following inequality holds:

+ + ≥ , , , the measures in radians.

Proposed by Marian Ursărescu – Romania

XI.25. ∈ (ℝ), det( + 3 + 3 ) = det( + 4 ) = 0. Find: Ω = det

Proposed by Marian Ursărescu – Romania

XI.26. ∈ (ℝ), det = 1, = ( ), det( + ) = 0. Find:

Ω = ( ∗), ∗ - adjoint of , =

Proposed by Marian Ursărescu – Romania

XI.27. ∈ (ℤ), det( + + ) = det( + ). Find: Ω = , ∈ ℕ, ≥ 2

Proposed by Marian Ursărescu – Romania

XI.28. ∈ (ℝ), det( + 2 + 2 ) = det( + ) = 0. Find: Ω = det

Proposed by Marian Ursărescu – Romania

XI.29 If ∈ ℕ; ≥ 2; , ∈ (0,∞); ∈ 1, then:

1+ 2 − + 3 + + 4 >

1( + 3) ⋅

Proposed by D.M. Bătinețu-Giurgiu, Delia Popescu – Romania

XI.30. If > 0; ∈ 1, ; ∈ ℕ; ≥ 2; ∈ ℝ then:

≥ (2 − ) ; =

Proposed by D.M. Bătinețu-Giurgiu, Delia Goiceanu – Romania

XI.31. If = ∑ ( ) then find: Ω = lim → 1 + −( ) ⋅

Proposed by D.M. Bătinețu-Giurgiu, Doina Cristina Călina – Romania

Page 70: R. M. M. - 24 · 2019. 11. 29. · draga tĂtucu mariana-romania claudia nĂnuȚi-romania dan nedeianu-romania gabriela bondoc-romania ovidiu ticuȘi-romania romanian mathematical

Romanian Mathematical Society-Mehedinți Branch 2020

69 ROMANIAN MATHEMATICAL MAGAZINE NR. 24

XI.32. If ( ) ⊂ ℝ; lim → = ∈ ℝ; (∃) > 0 such that lim → ( − ) = ∈ℝ then find: Ω = lim → (1 − + )

Proposed by D.M. Bătinețu-Giurgiu, Aurel Chiriță – Romania

XI.33. If , , ∈ 0, then in Δ the following relationship holds:

( ) + ( ) + ( ) > 8√3 ; ( – area)

Proposed by D.M. Bătinețu-Giurgiu, Aurel Chiriță – Romania

XI.34. If , , > 0; , , ∈ 0, then:

( + ) tansin + sin +

( + ) tansin + sin +

( + ) tansin + sin ≥ 2 3 ( + + )

Proposed by D.M. Bătinețu-Giurgiu, Nicolae Oprea – Romania

XI.35. If in Δ , ≤ ≤ then the following relationship holds:

( + + ) + + ≤( + )

4 ⋅

Proposed by D.M. Bătinețu-Giurgiu, Cristina Miu – Romania

XI.36. If , , , , , > 0; ( ) ; ( ) ⊂ ℝ sequences such that:

= + ; = + ; ∈ ℕ then:

+ ≤

Proposed by D.M. Bătinețu-Giurgiu, Amelia Curcă Năstăselu – Romania

XI.37. If , , > 0; ∈ 0, then:

sin+

sin+

sin+ 3√

tan> 6√

Proposed by D.M. Bătinețu-Giurgiu, Neculai Stanciu – Romania

XI.38. If , , ∈ 0, then: + + > + +

Proposed by D.M. Bătinețu-Giurgiu, Daniel Sitaru – Romania

Page 71: R. M. M. - 24 · 2019. 11. 29. · draga tĂtucu mariana-romania claudia nĂnuȚi-romania dan nedeianu-romania gabriela bondoc-romania ovidiu ticuȘi-romania romanian mathematical

Romanian Mathematical Society-Mehedinți Branch 2020

70 ROMANIAN MATHEMATICAL MAGAZINE NR. 24

XI.39. If , , ∈ (0, ) then in Δ the following relationship:

1sin +

1sin +

1sin >

√3; ( ≥ 0)

Proposed by D.M. Bătinețu-Giurgiu, Daniel Sitaru – Romania

XI.40. If , ∈ ℕ; ≥ 2; , ∈ 0, ; (∀) ∈ 1, then:

+tansin >

( + 1)(∑ )∑

Proposed by D.M. Bătinețu-Giurgiu, Claudia Nănuți – Romania

XI.41. In Δ the following relationship holds:

( + − ) ≥(2 )

⋅ ⋅

Proposed by Daniel Sitaru – Romania

XI.42. In acute or right Δ the following relationship holds:

sin( ) +

sin( ) +

sin( ) >

9−

4( ) + ( ) + ( )

Proposed by Daniel Sitaru – Romania

XI.43. If 0 < < then:

sin+

tan sin+

tan sin+

tan> 8

Proposed by Daniel Sitaru – Romania

XI.44. In Δ the following relationship holds:

+ + + + + ≤ 4

Proposed by Daniel Sitaru – Romania

Page 72: R. M. M. - 24 · 2019. 11. 29. · draga tĂtucu mariana-romania claudia nĂnuȚi-romania dan nedeianu-romania gabriela bondoc-romania ovidiu ticuȘi-romania romanian mathematical

Romanian Mathematical Society-Mehedinți Branch 2020

71 ROMANIAN MATHEMATICAL MAGAZINE NR. 24

XI.45. Let be the th prime number. Find:

Ω = lim→

+ − 23 + 5 − 8

Proposed by Daniel Sitaru – Romania

XI.46. In Δ the following holds:

6 sin + 3 sin 2 + 2 sin 3 > 0

Proposed by Daniel Sitaru – Romania

XI.47. If 0 ≤ , , ≤ then:

(sin + sin + sin ) ≥ 6 8

Proposed by Daniel Sitaru – Romania

XI.48 Find: , , ⊂ ℝ, lim → = , lim → = , lim → = , , , ∈ℝ∗, , , ∈ ℕ∗

Proposed by Marian Ursărescu – Romania

XI.49. Find:

Ω = lim→

∑ ∑ ∑∑ ( )

Proposed by Marian Ursărescu – Romania

XI.50. Find:

Ω = lim→

tan⋅ ⋅√

Proposed by Marian Ursărescu – Romania

XI.51. If ∈ (ℝ), det ≠ 0, ( ) = 3 ( ) then: ( ) = 3 ⋅ det ⋅ ( )

Proposed by Marian Ursărescu – Romania

Page 73: R. M. M. - 24 · 2019. 11. 29. · draga tĂtucu mariana-romania claudia nĂnuȚi-romania dan nedeianu-romania gabriela bondoc-romania ovidiu ticuȘi-romania romanian mathematical

Romanian Mathematical Society-Mehedinți Branch 2020

72 ROMANIAN MATHEMATICAL MAGAZINE NR. 24

XI.52. If ∈ (ℝ), ( ∗) = 0, ∗ - adjoint of then:

(det ) + ( ) + + 1 ≥ 2 det + ⋅ det

Proposed by Marian Ursărescu – Romania

XI.53. , > 0, = , = + , ∈ ℕ, ∈ ℕ∗, ≥ 2. Find:

Ω = lim→

( )

Proposed by Marian Ursărescu – Romania

XI.54. If ∈ (ℝ), det = 1, det( + ) = 0 then: ( ∗) = (( )∗),

∗ - adjoint of , – trace of .

Proposed by Marian Ursărescu – Romania

XI.55. Let , , be positive real numbers such that: = ( = 1,2, … )

Prove that: − + − + − ≥ 3( )

( = 1,2, … )

Proposed by Kunihiko Chikaya-Japan

XI.56. ( ) = ( ) . Find:

Ω = lim→

⋅ ( )− 672

Proposed by Lazaros Zachariadis-Greece

XI.57. Evaluate the limit:

lim→

tan( − 1)− log(sin( ) + 1)

√ + 1 − 1

Proposed by John Horton Conway-France

XI.58 Solve for real numbers:

Page 74: R. M. M. - 24 · 2019. 11. 29. · draga tĂtucu mariana-romania claudia nĂnuȚi-romania dan nedeianu-romania gabriela bondoc-romania ovidiu ticuȘi-romania romanian mathematical

Romanian Mathematical Society-Mehedinți Branch 2020

73 ROMANIAN MATHEMATICAL MAGAZINE NR. 24

⎩⎪⎪⎨

⎪⎪⎧

= tan37 − 4 sin 7

10 + 95−

= 2019

Proposed by Urfan Aliyev-Azerbaijan

XI.59 Resolve:

2018 √ + + 2019 √ + + 1 2018 √ + + √ + + 2019√ + + 2019 √ + + 2018 2018 √ + + √ + + 2019

= ( + )( + )

≥ 1; , > 0. Proposed by Jhoaw Carlos-Bolivia

XI.60 In Δ the following relationship holds:

cot 2 cot 2 + tan 2 tan 2 + cot 2 cot 2 − tan 2 tan 2 ≥1064

27

Proposed by Daniel Sitaru – Romania

XI.61 If , , > 0 then:

+ + ≥+2 +

+2 +

+2

Proposed by Daniel Sitaru – Romania

XI.62 In acute Δ the following relationship holds:

sin ⋅ sin ⋅ sin > ⋅ ⋅ ⋅ cos 2 ⋅ cos 2 ⋅ cos 2

Proposed by Daniel Sitaru – Romania

XI.63 If 0 < < < < < then:

csc 2 ⋅ csc 2 ⋅ csc 2 <sinsin

Proposed by Daniel Sitaru – Romania

Page 75: R. M. M. - 24 · 2019. 11. 29. · draga tĂtucu mariana-romania claudia nĂnuȚi-romania dan nedeianu-romania gabriela bondoc-romania ovidiu ticuȘi-romania romanian mathematical

Romanian Mathematical Society-Mehedinți Branch 2020

74 ROMANIAN MATHEMATICAL MAGAZINE NR. 24

XI.64 Find = − , , ∈ ℝ, + ≠ 0 such that: − 3 + 4 − 3 + =

Proposed by Daniel Sitaru – Romania

All solutions for proposed problems can be finded on the http//:www.ssmrmh.ro which is the adress of Romanian Mathematical

Magazine-Interactive Journal.

12-CLASS-STANDARD

XII.1. If 0 < ≤ then:

1+

1+

1≥ √3 ⋅ log

Proposed by Daniel Sitaru – Romania

XII.2. If 0 < ≤ , : [ , ] → (0,1], – continuous then:

( ) + ( )( ) ( ) + ( ) ( ) + ( ) ≤ 2( − ) ( )

Proposed by Daniel Sitaru – Romania

XII.3. If ≥ 0 then:

( + 1) +( (1 − ) + 1)

( + 1) ≥ ( + 1)

Proposed by Daniel Sitaru – Romania

XII.4. Find:

Page 76: R. M. M. - 24 · 2019. 11. 29. · draga tĂtucu mariana-romania claudia nĂnuȚi-romania dan nedeianu-romania gabriela bondoc-romania ovidiu ticuȘi-romania romanian mathematical

Romanian Mathematical Society-Mehedinți Branch 2020

75 ROMANIAN MATHEMATICAL MAGAZINE NR. 24

1cos √tan + 2

Proposed by Abdul Mukhtar-Nigeria

XII.5. Find:

√cot (tan + 2)

Proposed by Abdul Mukhtar-Nigeria

XII.6. Find: tan

√tan (tan + 1)

Proposed by Abdul Mukhtar-Nigeria

XII.7. Find:

2 + 44 sin(2 )− 2 cos(2 ) + 3 cos( )− 3 cos(3 )8 + 9 sin( )− sin(3 )

Proposed by Nader Al Homsi-Amman-Jordan

XII.8. Find:

√1 + ln (1 + ln ) + ln√1 + ln (ln ) + ln + 1

Proposed by Nader Al Homsi-Amman-Jordan

XII.9. Find: sec (1 + sin )

+ sin − cos − 1

Proposed by Nader Al Homsi-Amman-Jordan

XII.10. Find: ∫ (csc sec ⋅ cos tan sin cot )

Proposed by Naren Bhandari-Nepal

XII.11. Find :ℝ → ℝ, which admits primitives with the property

( − ) ⋅ ( − ) = − ,∀ ∈ ℝ, where > 0 and is a primitive of .

Page 77: R. M. M. - 24 · 2019. 11. 29. · draga tĂtucu mariana-romania claudia nĂnuȚi-romania dan nedeianu-romania gabriela bondoc-romania ovidiu ticuȘi-romania romanian mathematical

Romanian Mathematical Society-Mehedinți Branch 2020

76 ROMANIAN MATHEMATICAL MAGAZINE NR. 24

Proposed by Marian Ursărescu – Romania

XII.12. If , > 0; < ; + = ∈ 0, then find:

( − + ) log(1 + tan ⋅ tan )

Proposed by D.M. Bătinețu-Giurgiu, Neculai Stanciu – Romania

XII.13. Find:

Ω = lim→

1log 1 +

Proposed by Vasile Mircea Popa – Romania

XII.14. Find:

Ω = lim→

⎛ arctan1

Proposed by Vasile Mircea Popa – Romania

XII.15. Find:

Ω = lim→

(3 + 1)

Proposed by Vasile Mircea Popa – Romania

XII.16. Find:

Ω = lim→

( + 1)

Proposed by Vasile Mircea Popa – Romania

XII.17. Find:

Page 78: R. M. M. - 24 · 2019. 11. 29. · draga tĂtucu mariana-romania claudia nĂnuȚi-romania dan nedeianu-romania gabriela bondoc-romania ovidiu ticuȘi-romania romanian mathematical

Romanian Mathematical Society-Mehedinți Branch 2020

77 ROMANIAN MATHEMATICAL MAGAZINE NR. 24

Ω = lim→

sin( )

Proposed by Vasile Mircea Popa – Romania

XII.18. Find:

Ω = lim→

⎛ ⋅ tan1

Proposed by Vasile Mircea Popa – Romania

XII.19. Find:

Ω = lim→

( + 1)

Proposed by Vasile Mircea Popa – Romania

XII.20. Find:

Ω = lim→

1 ⋅ ⋅( + )( + )( + )

Proposed by Daniel Sitaru – Romania

XII.21. ∗ = + + 0.5− [ + + 0.5], ∘ = + + 0.2 − [ + + 0.2]

Prove that: ([0,1),∗) ≅ ([0,1),∘) as abelian groups. [∗] - great integer function.

Proposed by Daniel Sitaru – Romania

XII.22. Find:

Ω( , ) = tan( ) tan( ) tan ( + ) , , > 0,0 < < 2( + )

Proposed by Daniel Sitaru – Romania

XII.23. Λ = { ⁄ ∈ ℝ, :ℝ → ℝ, ( ) = + }, ∗ =

Page 79: R. M. M. - 24 · 2019. 11. 29. · draga tĂtucu mariana-romania claudia nĂnuȚi-romania dan nedeianu-romania gabriela bondoc-romania ovidiu ticuȘi-romania romanian mathematical

Romanian Mathematical Society-Mehedinți Branch 2020

78 ROMANIAN MATHEMATICAL MAGAZINE NR. 24

Θ = { ⁄ ∈ ℝ∗, :ℝ → ℝ, ( ) = }, ∘ =

Prove that: (Λ,∗) ≇ (Θ,∘) as abelian groups.

Proposed by Daniel Sitaru – Romania

XII.24. ∗ = 2 , ∘ = 3

Find an isomorphism between abelian groups (0,∞),∗ and (0,∞),∘

Proposed by Daniel Sitaru – Romania

XII.25 Prove that if 0 < < then:

√1 +> ( − ) + ln

Proposed by Daniel Sitaru – Romania

XII.26. Find:

Ω =

⎛ cos 7 + 7 cos 5 + 21 cos 3 + 35 cos(4 + sin )(cos 6 + 6 cos 4 + 15 cos 2 + 10)

Proposed by Daniel Sitaru – Romania

XII.27. Find:

Ω = lim→

sin

Proposed by Marian Ursărescu – Romania

XII.28. If the equation:

+ + + 2 + + + 1 = 0, , , ∈ ℝ

has all roots real numbers then: | − | ≥ 8.

Proposed by Marian Ursărescu – Romania

Page 80: R. M. M. - 24 · 2019. 11. 29. · draga tĂtucu mariana-romania claudia nĂnuȚi-romania dan nedeianu-romania gabriela bondoc-romania ovidiu ticuȘi-romania romanian mathematical

Romanian Mathematical Society-Mehedinți Branch 2020

79 ROMANIAN MATHEMATICAL MAGAZINE NR. 24

XII.29. The equation:

+ + + + + + + − 1 = 0, , , , , ∈ ℝ

has all roots real numbers. Prove that: | + − | ≥ 16

Proposed by Marian Ursărescu – Romania

XII.30. Solve the following system:

⎩⎪⎪⎨

⎪⎪⎧ + + + =

5612

( ) + ( ) + ( ) + ( ) + ( ) + ( ) =89

12

( ) + ( ) + ( ) + ( ) =89

12

( ) =1

Proposed by Jhoaw Carlos-Bolivia

XII.30. Prove without softs:

⋅ 1 + <

Proposed by Kunihiko Chikaya-Japan

XII.31. If 0 < ≤ < then:

tancos ≥ log

1 + tan1 + tan

Proposed by Daniel Sitaru – Romania

XII.32. ℤ = { ⁄ ∈ ℤ}, ∈ ℤ. Prove that: (2ℤ ∩ 5ℤ ∩ 11ℤ, +) ≅ (3ℤ∩ 7ℤ ∩ 13ℤ, +) as abelian groups.

Proposed by Daniel Sitaru – Romania

XII.33. Find:

Page 81: R. M. M. - 24 · 2019. 11. 29. · draga tĂtucu mariana-romania claudia nĂnuȚi-romania dan nedeianu-romania gabriela bondoc-romania ovidiu ticuȘi-romania romanian mathematical

Romanian Mathematical Society-Mehedinți Branch 2020

80 ROMANIAN MATHEMATICAL MAGAZINE NR. 24

Ω =√1 −

89 + √89 + +

Proposed by D.M. Bătinețu – Giurgiu; Dan Grigore – Romania

XII.34. Find:

Ω =2 tan − tan

( + tan )

Proposed by D.M. Bătinețu – Giurgiu; Claudia Preda – Romania

XII.35. If ∈ 0, ; > 1; :ℝ → ℝ continuous and even function then find:

Ω = ( ) + 1

Proposed by D.M. Bătinețu – Giurgiu; Daniel Sitaru – Romania

XII.36. If > 0; , :ℝ → (0,∞),ℎ:ℝ → ℝ are continuous functions; , even and ℎ even function then prove that:

( )( ) + ( ) + ℎ ( ) + ℎ( )

=( )( )

Proposed by D.M. Bătinețu – Giurgiu; Clarisa Cavachi – Romania

All solutions for proposed problems can be finded on the http//:www.ssmrmh.ro which is the adress of Romanian Mathematical

Magazine-Interactive Journal.

UNDERGRADUATE PROBLEMS

Page 82: R. M. M. - 24 · 2019. 11. 29. · draga tĂtucu mariana-romania claudia nĂnuȚi-romania dan nedeianu-romania gabriela bondoc-romania ovidiu ticuȘi-romania romanian mathematical

Romanian Mathematical Society-Mehedinți Branch 2020

81 ROMANIAN MATHEMATICAL MAGAZINE NR. 24

U.1. Prove this sharp inequality:

2 +23 1 −

− 1 + 2 cos√

< 0

Proposed by K. Srinivasa Raghava – AIRMC – India

U.2. If

1+ ( ) + ( + ) + =

cosh( )cos( )

(−1)+ ( ) + ( + ) +

then show that = √2 + 3

Proposed by K. Srinivasa Raghava – AIRMC – India

U.3. Evaluate in closed – form:

sin ( )√

Proposed by K. Srinivasa Raghava – AIRMC – India

U.4. Prove that: 4

+1!− 1

3!+ 2

5!− 3

7!+ 4

9!− 5

11!+ ⋯ =

64(63− 7 )

Proposed by K. Srinivasa Raghava – AIRMC – India

U.5. Prove the following relationship:

⎛ log(tan( ))sin ( ) + cos ( )

⎞ =√3

− −√3

+23−

log(3)6 − log(2)

2√3

+log(3)√3

where ( ) is Poly – Logarithm

Proposed by K. Srinivasa Raghava – AIRMC – India

U.6. If, for any complex number , ( ) > 0

Page 83: R. M. M. - 24 · 2019. 11. 29. · draga tĂtucu mariana-romania claudia nĂnuȚi-romania dan nedeianu-romania gabriela bondoc-romania ovidiu ticuȘi-romania romanian mathematical

Romanian Mathematical Society-Mehedinți Branch 2020

82 ROMANIAN MATHEMATICAL MAGAZINE NR. 24

( ) =ln( )

+

then prove that: 3 ( ) + ( ) = 0 at the point = 2

Proposed by K. Srinivasa Raghava – AIRMC – India

U.7. Prove the relationship:

ln( ) sech( ) tanh( ) =13

1−4

+ ln( ) −11 ln(2)

60+

7 (3)+ 10 (−3)− 4 ln( )

( ) – is derivative of Zeta function, – Glaisher constant, – Euler’s Constant

Proposed by K. Srinivasa Raghava – AIRMC – India

U.8. Find:

Ω = lim→

⎛ (1 + 2 csc )(1 + 2 cot )

Proposed by Mohamed Arahman Jama-Somalia

U.9. Find:

logΓ( )

Γ(1 − ) logΓ( )

Γ(1 + )

Proposed by Obidah Al Sharafy-Yemen

U.10. Find:

Ω = lim→

⋅ log( + 1) ⋅ tan ( )

√1−

Proposed by Abdul Mukhtar-Nigeria

U.11. Find:

Page 84: R. M. M. - 24 · 2019. 11. 29. · draga tĂtucu mariana-romania claudia nĂnuȚi-romania dan nedeianu-romania gabriela bondoc-romania ovidiu ticuȘi-romania romanian mathematical

Romanian Mathematical Society-Mehedinți Branch 2020

83 ROMANIAN MATHEMATICAL MAGAZINE NR. 24

Ω = lim→

1+ 1 + log(1 + )

Proposed by Abdul Mukhtar-Nigeria

U.12. Find:

Ω = lim→

log ⋅ log(1 − ) ⋅ (1 + log(1 − ))

Proposed by Abdul Mukhtar-Nigeria

U.13.

⋅ log Γ( ) ⋅ log Γ(1 − ) = +log(2 )

+ +log(2 )

+(2)( + log(2 ))

⋅ +(2)⋅

Find: Ω = + + + + +

Proposed by Abdul Mukhtar-Nigeria

U.14. Find:

Ω =cos ⋅ cos(2 )

sin , ∈ 0, 4

Proposed by Abdul Mukhtar-Nigeria

U.15. Find:

Ω = lim→

⎛ loglog −log +

Proposed by Abdul Mukhtar-Nigeria

U.16. Prove the following closed form:

4‼‼!!

+(−1)

(4 + 3)! =1

2 √√√

1√2

− 4 + sin1√2

+ 4

Proposed by Naren Bhandari-Nepal

Page 85: R. M. M. - 24 · 2019. 11. 29. · draga tĂtucu mariana-romania claudia nĂnuȚi-romania dan nedeianu-romania gabriela bondoc-romania ovidiu ticuȘi-romania romanian mathematical

Romanian Mathematical Society-Mehedinți Branch 2020

84 ROMANIAN MATHEMATICAL MAGAZINE NR. 24

U.17. Prove that:

− 1( )

+ −1

( )

= (2)

where (. ) is the Euler totient function of ( ) is the number ∈ ℕ such that 1 ≤ < and g.c.d. ( , ) = 1.

Proposed by Naren Bhandari-Nepal

U.18. Prove that:

12 +

(−1)+ 1 −

(−1) (−1)+ 1 = ln 2

Proposed by Naren Bhandari-Nepal

U.19. Show that:

+ 1−√

= 3 , where ⌊⋅⌋ is the floor function.

Proposed by Naren Bhandari-Nepal

U.20. Prove the following:

1 + + + +( − 1)! ! =

75203

Proposed by Naren Bhandari-Nepal

U.21. Let ∑ ( ≥ 1) be a convergent series of real positive numbers. Prove that:

⋅ ⋅sin( )

⋅ 10 ⋅ ( − 1)! < +∞

Proposed by Artan Ajredini-Serbie

U.22. Prove that:

Page 86: R. M. M. - 24 · 2019. 11. 29. · draga tĂtucu mariana-romania claudia nĂnuȚi-romania dan nedeianu-romania gabriela bondoc-romania ovidiu ticuȘi-romania romanian mathematical

Romanian Mathematical Society-Mehedinți Branch 2020

85 ROMANIAN MATHEMATICAL MAGAZINE NR. 24

2 − 2 − (−1) ( + )( + )! =

11

Proposed by Srinivasa Raghava-AIRMC-India

U.23. Find:

Ω = lim→

log− √3 + 1

Proposed by Abdul Mukhtar-Nigeria

U.24. Let for any > 0

( ) = sin (1 + )

Prove the relationship:

1 − ( ) = cot√2

( )

Proposed by Srinivasa Raghava-AIRMC-India

U.25. ( ) = + log + ∑⋅( )!

. Prove that:

cos+ ( ) ≥ ( ) − ( ), 0 < ≤

Proposed by Daniel Sitaru – Romania

U.26. Prove:

lim→

1 + ( ) − 3 1 + ( ) ( ) + ( )

Γ( ) = 3 − 6 + 3 − 2

Proposed by Obidah Al Sharafy-Yemen

U.27.

Page 87: R. M. M. - 24 · 2019. 11. 29. · draga tĂtucu mariana-romania claudia nĂnuȚi-romania dan nedeianu-romania gabriela bondoc-romania ovidiu ticuȘi-romania romanian mathematical

Romanian Mathematical Society-Mehedinți Branch 2020

86 ROMANIAN MATHEMATICAL MAGAZINE NR. 24

=(2 − 1)‼(2 + 2)‼

Find:

Ω = lim→

( + ) − ( )

Proposed by Daniel Sitaru – Romania

U.28. Evaluate:

2log(tan )

where, ( ) are Euler Polynomials

Proposed by Arafat Rahman Akib-Bangladesh

U.29. Find the limit:

lim→

1 +1

10 ⋅ !

Proposed by Naren Bhandari-Nepal

U.30. Find:

lim→

arccos( )1

tan ln1

cos

Proposed by Abdul Mukhtar-Nigeria

U.31.Find:

lim→

arcsec( ) ln(1 − )2 − 2 + 1

Proposed by Abdul Mukhtar-Nigeria

Page 88: R. M. M. - 24 · 2019. 11. 29. · draga tĂtucu mariana-romania claudia nĂnuȚi-romania dan nedeianu-romania gabriela bondoc-romania ovidiu ticuȘi-romania romanian mathematical

Romanian Mathematical Society-Mehedinți Branch 2020

87 ROMANIAN MATHEMATICAL MAGAZINE NR. 24

U.32. Find:

−1

1 + +1 − coth( )

Proposed by Abdul Mukhtar-Nigeria

U.33. Find:

ln√1 + cos

cos

Proposed by Abdul Mukhtar-Nigeria

U.34. Prove that:

ln ( )( )

cos( )

(1 + sin( )) 1− sin( )= −8

Proposed by Nader Al Homsi- Jordan

U.35.

…2

2 − 1 … = 1

Find the value of such that above equality holds true, where ⌊⋅⌋ denotes the floor function.

Proposed by Naren Bhandari-Nepal

U.36. Prove:

Ω =( ) ⋅ sin( )

= − { + log(2)}

Proposed by Obidah Al Sharafy- Yemen

U.37. Prove:

Ω = { + (1 + )} =32

Page 89: R. M. M. - 24 · 2019. 11. 29. · draga tĂtucu mariana-romania claudia nĂnuȚi-romania dan nedeianu-romania gabriela bondoc-romania ovidiu ticuȘi-romania romanian mathematical

Romanian Mathematical Society-Mehedinți Branch 2020

88 ROMANIAN MATHEMATICAL MAGAZINE NR. 24

where ( ) DiGamma function

Proposed by Obidah Al Sharafy- Yemen

U.38. Prove:

Ω =( ) Γ( )

Γ( ) −( ) ( )

( )

−1 + Γ( ) −1 + ( )( )

=

=1

√ ( − 1)+

12√ ( − 1)

Where ( )DiGamma function, ( ) Polylogarithmic, =

, =

Proposed by Obidah Al Sharafy-Yemen

U.39. Prove:

Ω =cos( ( ) + )

{ + ( ( ) + )} =(1 + )2

where ( ) polyGamma function

Proposed by Obidah Al Sharafy- Yemen

U.40. Find:

Ω =log( + 1)+ + 1

Proposed by Vasile Mircea Popa – Romania

U.41. Find:

Ω =⋅ log( + 1)

+ + 1

Proposed by Vasile Mircea Popa – Romania

Page 90: R. M. M. - 24 · 2019. 11. 29. · draga tĂtucu mariana-romania claudia nĂnuȚi-romania dan nedeianu-romania gabriela bondoc-romania ovidiu ticuȘi-romania romanian mathematical

Romanian Mathematical Society-Mehedinți Branch 2020

89 ROMANIAN MATHEMATICAL MAGAZINE NR. 24

U.42. Find:

Ω( ) = ( + + 1)(1 + ) , > 0

Proposed by Vasile Mircea Popa – Romania

U.43. Find:

Ω( ) = ( − + 1)(1 + ) , > 0

Proposed by Vasile Mircea Popa – Romania

U.44. Find:

Ω = lim→

1− 1

arcsin

Proposed by Vasile Mircea Popa – Romania

U.45. Find: Ω( ) = ∫ ( )( ), > 0

Proposed by Vasile Mircea Popa – Romania

U.46. If , ∈ ℝ then:

8 (cos cos cos( + )) + ( − ) ≥ 0

Proposed by Daniel Sitaru – Romania

U.47. Calculate:

arctan( )

√1 +−

12

arctan( )

√1 +−

1√1 −

arctanh1 −1 +

Proposed by Cornel Ioan Vălean – Romania

U.48. Ω( , , ) = ∫⋅( ) ⋅

( ⋯ ), , , ∈ ℕ∗. Find:

Page 91: R. M. M. - 24 · 2019. 11. 29. · draga tĂtucu mariana-romania claudia nĂnuȚi-romania dan nedeianu-romania gabriela bondoc-romania ovidiu ticuȘi-romania romanian mathematical

Romanian Mathematical Society-Mehedinți Branch 2020

90 ROMANIAN MATHEMATICAL MAGAZINE NR. 24

Ω(17,02,2008)

Proposed by Feti Sinani-Kosovo

U.49. Find:

Ω = lim→

log(1 + 2 )

Proposed by Max Wong-Hong Kong

U.50. Find:

sin(ln ) ( )

Proposed by Nawar Alasadi-Iraq

U.51. If 0 < ≤ then:

( + )( + )( + )

≥( + )( − )

2 3 cos 7 − cos37

Proposed by Daniel Sitaru – Romania

U.52. If 0 < ≤ < then:

8sin ⋅ sin ⋅ sin( + )

( − − ) ≤ (3 − 3 )√3

Proposed by Daniel Sitaru – Romania

U.53. If 0 < ≤ < 2 then:

16( − )

sin sin sin

sin sin sin sin+ cot

+ +2

≤ log2 sin

Proposed by Daniel Sitaru – Romania

Page 92: R. M. M. - 24 · 2019. 11. 29. · draga tĂtucu mariana-romania claudia nĂnuȚi-romania dan nedeianu-romania gabriela bondoc-romania ovidiu ticuȘi-romania romanian mathematical

Romanian Mathematical Society-Mehedinți Branch 2020

91 ROMANIAN MATHEMATICAL MAGAZINE NR. 24

All solutions for proposed problems can be finded on the http//:www.ssmrmh.ro which is the adress of Romanian Mathematical

Magazine-Interactive Journal.

ROMANIAN MATHEMATICAL MAGAZINE-R.M.M.-autumn 2019

PROBLEMS FOR JUNIORS

JP.196. Let , , be the sides in a triangle such that = 1. Find the minimum value of:

=√ + − 1

+√ + − 1

+√ + − 1

+3( + + )

+ +

When does it occurs?

Proposed by Hoang Le Nhat Tung – Hanoi – Vietnam

JP.197. Solve for real numbers:

6 2 − 2 + 1 + 4 3 − 2 = 2 − 5 + 13

Proposed by Hoang Le Nhat Tung – Hanoi – Vietnam

JP.198. Prove that in any Δ the following inequality holds:

min( , , ) ≤ 4 ( + ) ≤ max( , , )

Proposed by Marian Ursărescu – Romania

JP.199. Let be a pyramid with the base parallelogram and any point which bolongs to the side such that: = . Through the vertex and the point we consider a variable plane which intersects the segment in and the segment in . Prove that:

≥2+ 1

Proposed by Marian Ursărescu – Romania

JP.200. Let be :ℝ → ℝ such that: ( ) + ( ) ≥ 2 ; (∀) , ∈ ℝ. Prove that:

( ) + ( ) + ⋯+ ( ) ≥+ + ⋯+

; (∀) ≥ 2;

Page 93: R. M. M. - 24 · 2019. 11. 29. · draga tĂtucu mariana-romania claudia nĂnuȚi-romania dan nedeianu-romania gabriela bondoc-romania ovidiu ticuȘi-romania romanian mathematical

Romanian Mathematical Society-Mehedinți Branch 2020

92 ROMANIAN MATHEMATICAL MAGAZINE NR. 24

(∀) , , … , ∈ ℝ.

Proposed by D.M. Bătinețu-Giurgiu, Daniel Sitaru – Romania

JP.201. If , , > 0 then:

( + )+ + 2 +

( + )+ + 2 +

( + )+ + 2 ≥ 2 3 ( + + )

Proposed by D.M. Bătinețu-Giurgiu, Neculai Stanciu – Romania

JP. 202. Let , , be positive real numbers such that + + + 2 = 1.

Prove that:

√2 + 16 + 7+√2 + 16 + 7

+√2 + 16 + 7

≥3

20

Proposed by Hoang Le Nhat Tung – Hanoi – Vietnam

JP.203. If , , > 0; ⋅ ⋅ = 1 then:

− √ + − √ + − √ ≥ 0

Proposed by Daniel Sitaru – Romania

JP.204. In Δ the following relationship holds:

cos cos

tan+

cos cos

tan+

cos cos

tan>

Proposed by Hoang Le Nhat Tung – Hanoi – Vietnam

JP.205. Let , , be positive real numbers. Prove that:

++

23

++

23

++

23 ≥

83

Proposed by George Apostolopoulos – Messolonghi – Greece

JP.206. Let be a triangle with inradius and circumradius . Let ℎ ,ℎ , ℎ the altitudes to sides , , respectively and let , , the exradii to , , respectively. Prove that:

Page 94: R. M. M. - 24 · 2019. 11. 29. · draga tĂtucu mariana-romania claudia nĂnuȚi-romania dan nedeianu-romania gabriela bondoc-romania ovidiu ticuȘi-romania romanian mathematical

Romanian Mathematical Society-Mehedinți Branch 2020

93 ROMANIAN MATHEMATICAL MAGAZINE NR. 24

4≤

ℎ⋅ +

ℎ⋅ +

ℎ⋅ ≤ 2

Proposed by George Apostolopoulos – Messolonghi – Greece

JP.207. Let , , be the lengths of the sides of a triangle with inradius and circumradius , and let , , the exradii to , , respectively. Prove that:

6 ≤ + + + + + ≤2 −

Proposed by George Apostolopoulos – Messolonghi – Greece

JP.208. Prove that in any triangle the following inequality holds:

tan + tan≤

Proposed by Marin Chirciu – Romania

JP.209. If , , , ∈ ℝ then:

+ + | − | ≤ 2( + )( + )

Proposed by Daniel Sitaru – Romania

JP.210. Let , , be positive real numbers such that + + = 3. Prove that:

+ +9 ≤

1+ ( + ) +

1+ ( + ) +

1+ ( + ) ≤

13

Proposed by George Apostolopoulos – Messolonghi – Greece

PROBLEMS FOR SENIORS

SP.196. Find:

lim→

1, ∈ ℕ∗, ≥ 2

Proposed by Marian Ursărescu – Romania

Page 95: R. M. M. - 24 · 2019. 11. 29. · draga tĂtucu mariana-romania claudia nĂnuȚi-romania dan nedeianu-romania gabriela bondoc-romania ovidiu ticuȘi-romania romanian mathematical

Romanian Mathematical Society-Mehedinți Branch 2020

94 ROMANIAN MATHEMATICAL MAGAZINE NR. 24

SP.197. If , , ≥ 0 then:

7√

5√ + 3√+

7√

5√ + 3√+

7√

5à + 3ò

√7√5 + √3

+√7

√5 + √3+

√7√5 + √3

Proposed by Daniel Sitaru – Romania

SP.198. If , , , ∈ ℝ; + = + = 10 then:

(10 − − 3 )(10 − − )(10 − − 3 ) < 10125

Proposed by Daniel Sitaru – Romania

SP.199. If > 1, ∈ ℕ then:

1log 2

2 − 1<

1 ⋅ 3 ⋅ 7 ⋅… ⋅ (2 − 1)(2 )!

Proposed by Daniel Sitaru – Romania

SP.200. If , , , ∈ ℝ then:

2| − |( + ) + ( + ) ≤ ( − ) + ( + )( + )√2

Proposed by Daniel Sitaru – Romania

SP.201. Find:

Ω = lim→

tan1

2( + 1) tan2 + 4 + 1

2( + 1)

Proposed by Daniel Sitaru – Romania

SP.202. Prove that in any triangle , the following relationship holds:

+ + ≥ 3 +−+ +

−+ +

−+

Proposed by Nguyen Viet Hung – Hanoi – Vietnam

SP.203. Let , , be positive real numbers such that ( + )( + )( + ) = 8. Prove that:

Page 96: R. M. M. - 24 · 2019. 11. 29. · draga tĂtucu mariana-romania claudia nĂnuȚi-romania dan nedeianu-romania gabriela bondoc-romania ovidiu ticuȘi-romania romanian mathematical

Romanian Mathematical Society-Mehedinți Branch 2020

95 ROMANIAN MATHEMATICAL MAGAZINE NR. 24

1+ + +

1+ + ≥

23

Proposed by Nguyen Viet Hung – Hanoi – Vietnam

SP.204. Let , , be positive real numbers such that + + = 3. Prove that:

+ 2 + + 2 + + 2 ≥ 1

Proposed by Nguyen Viet Hung – Hanoi – Vietnam

SP.205. In Δ , , , are length’s of Nagel’s cevians. Prove that:

≥ , , , – exradii of triangle.

Proposed by Daniel Sitaru – Romania

SP.206. Prove that in any triangle the following inequality holds:

−2 + 17 ≤ tan 2 ≤6

( − 5 )

Proposed by Marin Chirciu – Romania

SP.207. Prove that in any triangle the following inequality holds:

9(8 − 23 ) ≤ cot 2 ≤8132

(13 − 88 )

Proposed by Marin Chirciu – Romania

SP.208. Prove that in any triangle the following inequality holds:

36 ≤ sec 2 ≤ 9

Proposed by Marin Chiricu – Romania

SP.209. Prove that in any triangle the following inequality holds:

27 ≤ csc 2 ≤4

(4 − 37 )

Proposed by Marin Chiricu – Romania

SP.210. Let be an acute-angled triangle. If + + = and

Page 97: R. M. M. - 24 · 2019. 11. 29. · draga tĂtucu mariana-romania claudia nĂnuȚi-romania dan nedeianu-romania gabriela bondoc-romania ovidiu ticuȘi-romania romanian mathematical

Romanian Mathematical Society-Mehedinți Branch 2020

96 ROMANIAN MATHEMATICAL MAGAZINE NR. 24

cos + cos + cos = ; ( , , – the measures in radians), then Δ is equilateral.

Proposed by Marian Ursărescu – Romania

UNDERGRADUATE PROBLEMS

UP.196. Let be , > 0, ≠ such that lim → = lim → = , ∈ ℕ∗. Find:

lim→

−−

Proposed by Marian Ursărescu – Romania

UP.197. Let be :ℝ → (0,∞) continuous such that for , , > 0, fixed values

( ) + ( ) + ( ) = ( ) ( ) ( ), (∀) , , ∈ ℝ

Prove that:

( ) ≥( − )( + + )√ + +

3 ; (∀)0 < ≤

Proposed by Daniel Sitaru – Romania

UP.198. Let be a positive integer. Evaluate:

lim→

1 − (cos ) cos( )

Proposed by Nguyen Viet Hung – Hanoi – Vietnam

UP.199. Given the triangle . The internal angle bisectors from , , meet sides , , at , , respectively. Prove that:

tan2

+ tan2

+ tan2

+cos ,

cos+

cos ,

cos+

cos ,

cos= 0

Proposed by Nguyen Viet Hung – Hanoi – Vietnam

UP.200. If 0 < ≤ then:

Page 98: R. M. M. - 24 · 2019. 11. 29. · draga tĂtucu mariana-romania claudia nĂnuȚi-romania dan nedeianu-romania gabriela bondoc-romania ovidiu ticuȘi-romania romanian mathematical

Romanian Mathematical Society-Mehedinți Branch 2020

97 ROMANIAN MATHEMATICAL MAGAZINE NR. 24

( + + + )+ + √ + √

≤( + ) ( − )

4

Proposed by Daniel Sitaru – Romania

UP.201. Calculate the integral:

arctan− + 1

It is required to express the integral value with the usual mathematical constants and Ψ , where Ψ ( ) is the trigamma function.

Proposed by Vasile Mircea Popa – Romania

UP.202. Prove that:

Ψ5

12 =32− 6√3

3 + 40 − 10Ψ13

Ψ1112 =

32 + 6√33 − 40 − 10Ψ

13

where Ψ ( ) is the trigamma function and is the Catalan’s constant.

Proposed by Vasile Mircea Popa – Romania

UP.203. Given a triangle with incenter . The lines , , meet the sides , , at , , and meet the circumcircle at the second points , , respectively. Prove that:

(a) + + = 2,

(b) + + = − 1

Proposed by Nguyen Viet Hung – Hanoi – Vietnam

UP.204. Let ( ) be a positive real sequence such that lim → = ∈ ℝ∗ , where

is a positive integer. Compute:

lim→

1[ ⋅ ]

Page 99: R. M. M. - 24 · 2019. 11. 29. · draga tĂtucu mariana-romania claudia nĂnuȚi-romania dan nedeianu-romania gabriela bondoc-romania ovidiu ticuȘi-romania romanian mathematical

Romanian Mathematical Society-Mehedinți Branch 2020

98 ROMANIAN MATHEMATICAL MAGAZINE NR. 24

where ∈ ℝ; we denote by [ ] the integer part of .

Proposed by D.M. Bătinețu-Giurgiu, Neculai Stanciu – Romania

UP.205. Compute:

lim→

lim→

Γ( + 2) ( ) − Γ( + 1)

where ( ) , = 0, = 1, = + ,∀ ∈ ℕ is the Fibonacci sequence.

Proposed by D.M. Bătinețu-Giurgiu, Neculai Stanciu – Romania

UP.206. Compute:

lim→

lim→

Γ( + 2) ( ) − Γ( + 1) ,

where ( ) , = 0, = 1, = + ,∀ ∈ ℕ is the Fibonnaci sequence.

Proposed by D.M. Bătinețu-Giurgiu, Neculai Stanciu – Romania

UP.207. Let be ∈ (ℝ) such that det = −1. Prove that:

( + + 1) ≥ 3( ⋅ − 1)

Proposed by Marian Ursărescu – Romania

UP.208. Let be an acute – angled triangle and , , the points in which the heights of the triangle intersect the circumcircle of Δ . Prove that:

≤2

Proposed by Marian Ursărescu – Romania

UP.209. Demonstrate the followig inequality:

+ + + + ⋯+ + ≤ + 1

where , , … , are strictly positive real numbers which satisfy the relationship

+ + ⋯+ =

Proposed by Vasile Mircea Popa – Romania

Page 100: R. M. M. - 24 · 2019. 11. 29. · draga tĂtucu mariana-romania claudia nĂnuȚi-romania dan nedeianu-romania gabriela bondoc-romania ovidiu ticuȘi-romania romanian mathematical

Romanian Mathematical Society-Mehedinți Branch 2020

99 ROMANIAN MATHEMATICAL MAGAZINE NR. 24

UP.210. Prove that for any acute triangle the following inequality holds:

cot + cot + cot + √3 ≥ 2 tan 2 + tan 2 + tan 2

Proposed by Vasile Mircea Popa – Romania

All solutions for proposed problems can be finded on the http//:www.ssmrmh.ro which is the adress of Romanian Mathematical

Magazine-Interactive Journal.

Page 101: R. M. M. - 24 · 2019. 11. 29. · draga tĂtucu mariana-romania claudia nĂnuȚi-romania dan nedeianu-romania gabriela bondoc-romania ovidiu ticuȘi-romania romanian mathematical

Romanian Mathematical Society-Mehedinți Branch 2020

100 ROMANIAN MATHEMATICAL MAGAZINE NR. 24

INDEX OF AUTHORS RMM-24

Nr.crt. Numele și prenumele Nr.crt. Numele și prenumele 1 DANIEL SITARU-ROMANIA 42 SRINIVASA RAGHAVA-INDIA 2 D.M.BĂTINEȚU-GIURGIU-ROMANIA 43 NGUYEN VAN NHO-VIETNAM 3 CLAUDIA NĂNUȚI-ROMANIA 44 HUNG NGUYEN VIET-VIETNAM 4 NECULAI STANCIU-ROMANIA 45 MOHAMED ARAHMAN JAMA-SOMALIA 5 MARIAN URSĂRESCU-ROMANIA 46 ARAFAT RAHMAN AKIB-BANGLADESH 6 BOGDAN FUSTEI-ROMANIA 47 OBIDAH AL SHARAFY-YEMEN 7 DAN NĂNUȚI-ROMANIA 48 GEORGE APOSTOLOPOULOS-GREECE 8 MARIN CHIRCIU-ROMANIA 49 HOANG LE NHAT TUNG-VIETNAM 9 TITU ZVONARU-ROMANIA 50 MEHMET SAHIN-TURKEY

10 TUTESCU LUCIAN-ROMANIA 51 ROVSEN PIRGULIEV-AZERBAIJAN 11 PETRE STĂNGESCU-ROMANIA 52 NADER AL HOMSI-JORDAN 12 VASILE MIRCEA POPA-ROMANIA 53 ADIL ABDULLAYEV-AZERBAIJAN 13 LUIZA CREMENEANU-ROMANIA 54 IOAN CORNEL VALEAN-ROMANIA 14 IULIANA TRAȘCĂ-ROMANIA 55 NAWAR ALASADI-IRAQ 15 CAMELIA DANĂ-ROMANIA 56 MARTIN LUKAREVSKI-MACEDONIA 16 LAVINIA TRINU-ROMANIA 57 MAX WONG-HONG KONG 17 ALINA ȚIGAE-ROMANIA 58 KUNIHIKO CHIKAYA-JAPAN 18 SIMONA RADU-ROMANIA 59 SEYRAN IBRAHIMOV-AZERBAIJAN 19 SIMONA MIU-ROMANIA 60 JHOAW CARLOS-BOLIVIA 20 BETIU ANICUTA-ROMANIA 61 NAREN BHANDARI-NEPAL 21 CĂTĂLINA PANĂ-ROMANIA 62 URFAN ALIYEV-AZERBAIJAN 22 GABRIEL TICĂ-ROMANIA 63 FETI SINANI-KOSOVO 23 VIRGINIA GRIGORESCU-ROMANIA 64 ABDUL MUKHTAR-NIGERIA 24 CLAUDIU CIULCU-ROMANIA 65 ARTAN AJREDINI-SERBIE 25 MIHAELA STĂNCELE-ROMANIA 66 CLARISA CAVACHI-ROMANIA 26 RAMONA NĂLBARU-ROMANIA 67 CLAUDIA PREDA-ROMANIA 27 OANA PREDA-ROMANIA 68 DAN GRIGORE-ROMANIA 28 CONSTANTINA PRUNARU-ROMANIA 69 OLEG ȚURCAN-PORTUGAL 29 TATIANA CRISTEA-ROMANIA 70 CRISTIAN MOANȚĂ-ROMANIA 30 DOINA CRISTINA CĂLINA-ROMANIA 71 ALEXANDRINA NĂSTASE-ROMANIA 31 SANDA IULIA-ROMANIA 72 ILEANA DUMA-ROMANIA 32 ROXANA VASILE-ROMANIA 73 CONSTANTIN BASARAB-ROMANIA 33 VASILE BURUIANĂ-ROMANIA 74 MARIAN VOINEA-ROMANIA 34 OCTAVIAN STROE-ROMANIA 75 ALECU ORLANDO-ROMANIA 35 DANA COTFASĂ-ROMANIA 76 NGUYEN VAN CANH-VIETNAM 36 SORIN PĂRLEA-ROMANIA 77 MUSTAFA TAREK-EGYPT 37 ȘTEFAN MARICA-ROMANIA 78 THANASIS GAKOPOULOS-GREECE 38 CARINA MARIA VIESPESCU-ROMANIA 79 PEDRO PANTOJA-BRAZIL 39 DELIA POPESCU-ROMANIA 80 AUREL CHIRIȚĂ-ROMANIA 40 DELIA GOICEANU-ROMANIA 81 NICOLAE OPREA-ROMANIA 41 CRISTINA MIU-ROMANIA 82 AMELIA CURCĂ NĂSTĂSELU-ROMANIA

NOTĂ: Pentru a publica probleme propuse, articole și note matematice în RMM puteți trimite materialele pe mailul: [email protected]